Bdtmathlinks PDF
Bdtmathlinks PDF
Prove that:
a2
a+b+c
b2
c2
+
+
a2 + bc b2 + ca c2 + ab
2 3 abc
2, Let a, b and c are positive numbers. Prove that:
5 ab + ac + bc
a2
b2
c2
2
+
+
2 a + b2 + c2
a2 + bc b2 + ca c2 + ab
3, Given a, b, c 0. Prove that:
a2
b2
c2
(a + b + c)2
2
+ 2
+ 2
2(ab + bc + ca)
a + bc b + ca c + ab
HD
C1
X a2
X
2a2
a2 (a b)(a c)
=
0
2
(a + b)(a + c)
a + bc
(a + b)(a + c)(a2 + bc)
P
X
(a + b + c)2
2 ab(a + b)
2a2
=
2(ab + bc + ca)
(a + b)(a + c)
(a + b)(b + c)(c + a)
4q
qr
qr
4q
q 3r
2r
4q 1
q 3r
4q1
9 ,
then
2r
6r
2r
=
q 3r
1
q
q 4q1
3
It suffices to show that
6r (4q 1)(1 q)
But this is just Schurs Inequality for fourth degree
X
X
X
a4 + abc
a
ab(a2 + b2 )
We have done.
C2
My proof: Suppose a + b + c = 3. We need to prove:
f (r) = 4q 4 9q 3 + 24qr2 54q 2 r 72r2 243r + 216qr 0
1
9
4 , f (r)
f (r) f (
4q 9
) 0(trues with q
3
9
4,
we have:
9
4)
3
9
3
P 2 2
P 2
P 2 = P 2 2
2
2
2
2(a + b + c) 5 a
(a + b + c )(6 a 5 a )
( a )
1
((a2 + 3)(b2 + 3)(c2 + 3))2 64
64
Note : Its better if you think more about classical ineq before use modern tech 6, Find
the best value of k to this ineq is truefor all a, b, c 0, abc = 1
X a2 + b2
X ab 3 X c2 X a + b
k(
+
)
+
c2
c2
4
ab
2c
I think it can be killed by pqr tech but it is not a nice proof.
with a = b = c we find k = 58 Let a + b + c = p, ab + bc + ca = q, abc = r. We have:
Ineq
5 X 2 2 2 2 X 3 3 3a2 b2 c2
1X
a b (a +b )+
a b
abc(a3 +b3 +c3 )+
ab(a+b)
8
4
2
5 2 2 2 2 2 2 2 2 2 X 3 3 3a2 b2 c2
1X
15
(a +b +c )(a b +b c +c a )+
a b
abc(a3 +b3 +c3 )
ab(a+b)
0
8
4
2
8
5
1
21
(p2 2q)(q 2 2p) + (q 3 3pq + 3) (p3 3pq + 3) (pq 3)
0
8
2
8
5p2 q 2
q3
9p3
9
+ 2pq 0
8
4
4
8
Follow Schur:
r
C2
(4q p2 )(p2 q)
(4q p2 )(p2 q)
1
6p
6p
X ab 3 X c2 X a + b
5 X a2 + b2
)
+
(
+
8
c2
c2
4
ab
2c
2 2
3
3
5p q 10p 10q + 16pq 9 0
setting
f (p) = 5p2 q 2 10p3 10q 3 + 16pq 9
f 0 (p) = 10pq 2 30p2 + 16q 0
p
1
((a2 + 3)(b2 + 3)(c2 + 3))2 64
64
Note : Its better if you think more about classical ineq before use modern tech.
C2 I proven the problem of tranvanluan but I didnt use your the problem Wink this
my proof.
Lemma:
q 2 (1 q)
r
2(2 3q)
3
and
r
(4q 1)(1 q)
6
r
a2 + b2
a2 + b2
f (a; b; c) f (
;
; c)
2
2
p
a2 + b2
c(a2 + b2 )
) + [c(a + b) c 2(a2 + b2 )] [abc
]
= (ab
2
2
(a b)2
c(a b)2
c(a b)2
p
=
+
2
2
a + b + 2(a2 + b2 )
r
c 1
c
p
= (a b)2 (
)0
2 2 a + b + 2(a2 + b2 )
r
r
a2 + b2
a2 + b2
;
; c)
f (a; b; c) f (
2
2
f (a; b; c) f (t; t; t) = 2
4
C2 Note:
3
2(abc + 2) = a2 + b2 + c2 + 2abc + 1 a2 + b2 + c2 + 3 a2 b2 c2
9, Let a1 , a2 , a3 , ..., an be n non-negative real numbers, such that a1 +a2 +.....an = 1.
Prove that
1
a1 a2 + a2 a3 + a3 a4 + ...........an1 an
4
C1 It does not work when n = 1. You need n 2. When n = 2, and a1 + a2 = 1, we
do have a1 a2 14 . The proof is easy and Ill omit it here.
Assume that for some n 2, a1 a2 + +an1 an 14 whenever a1 + +an = 1. Let
a1 +a2 + +an +an+1 = 1. WLOG, assume that an an1 . Then if A = an +an+1
we have a1 + + an1 + A = 1; hencea1 a2 + + an1 A 14 .
a1 a2 + + an1 an + an an+1 a1 a2 + + an1 (an + an+1 )
1
.
4
1
4
+1 y +1 z +1
2
x4
Let
f (x, y, z) =
Then
x3
y3
z3
3
+
+
x4 + 1 y 4 + 1 z 4 + 1 2
f (x, y, z) f (x, yz, yz) 0
But
f (x,
0
yz, yz) = f ( 2 , t, t) =
t
2(t8 + 1)(t4 + 1)
3
a3 b
a4 + b4
2
we have:
2LHS
X
cyc
X
X 2a8
X
2a3 b
a3 b
q
rq
=
( 8
)1/4
8
8
8
a +b
a8 +b8
cyc
cyc
cyc
a2 b2 + a +b
2
ab
2
2a4
3 = 2RHS
+ b4
a4
the last inequality is true since its Vascs Mr. Green ,and I saw somewhere on this forum , two nice proofs with cauchy and Jensen Smile
11, Let x, y, z are non-negative numbers which not two of them equal to 0. Prove that:
r
r
r
r
x
y
z
xyz
+
+
2 1+
y+z
x+z
y+x
(x + y)(x + z)(y + z)
<=>
x(x + y)(x + z) + 2
X
X
p
(x + y) xy(x + z)(y + z) 4(
x)(
xy)
X
X
X
X
xy(x+y)4(
x)(
xy) =
x3 +3xyz
xy(x+y) 0
ab =
1 q2
, abc = r
3
We will prove
<=>
Use
r
1 + q)2 (1 2q)
27
4q 1
1
)=
(q 1)(3q 1)(9q 2 8q + 2) 0
9
27
6
Done Smile
13, Let a, b, c > 0 Prove that:
27(a3 b3 +b3 c3 +c3 a3 )2 (a+b+c)(ab+c)(a+bc)(a3 +b3 +c3 )2 (a+b+c)3
14, Let a, b, c > 0. Prove that
(ab + bc + ca)3 (a + b + c)(a b + c)(a + b c)(a + b + c)3
Assume p = 1
4q 1
9
<=> (3q 1)(3q 2 + q 1) 0
r
x5
y5
z5
3(x6 + y 6 + z 6 )
+ 2
+ 2
2
2
2
+z
x +z
y +x
2(x3 + y 3 + z 3 )
2
2
2
y2 + z2
2 ( x +y3 +z )3
Nice! Very Happy. I have proved it by the following idea:
X
cyc
y2
x5
(x6 + y 6 + z 6 )2
3(x6 + y 6 + z 6 )
P 7 2
2
7
2
+z
(x y + x z )
2(x3 + y 3 + z 3 )
Setting : x + y = z t = k
=> x7 + (k x)7 = (k + t)7 t7
<=> k(x + t)(t + x k) = 0
<=> (t + z)(t + y)(t + x) = 0
=> S = 0
20, For a, b, c > 0. Prove that
a3
b3
c3
3.(ab + bc + ca)
+
+
2
2
2
2
2
2
b + c bc c + a ac a + b bc
a+b+c
P 2 2
( a )
LHS P
ab(a + b) 3abc
Assume:
4q 1
9
<=> (3q 1)2 0
p = 1; r
abc
a2 + b2 + c2
4
+
3
3
+b +c
ac + ab + bc
3
Assume p = 1
<=> (3q 1)(10q 9r 3) 0
By schur we have:
9r 4q 1
(3q 1)(10q 9r 3) 2(3q 1)2 0
Done Smile
X
(a b)2 (
1
a+b+c
)0
ab + bc + ca 3(a3 + b3 + c3 )
2
2
p
p
X p
X p
a a2 ab + b2 a2 ac + c2 =
a3 a2 b + b2 a a3 a2 c + c2 a (a3 +b3 +c3 )2
cyclic
cyclic
(a3 a2 b + b2 a)(a3 a2 c + c2 a)
cyc
1X 3 2
(a a b+b2 a+a3 a2 c+c2 a) == a3 +b3 +c3
2 cyc
(a + b)2
3
c(2c + a + b)
Let a + b + c = 3. Then
X
cyc
X
(a + b)2
3
c(2c + a + b)
cyc
(3 c)2
1 0
c(3 + c)
X 1 c
X (c 1)2 (4 + c)
X 1c
c1
0
+
0
0
c(3 + c)
c(3 + c)
4
c(3 + c)
cyc
cyc
cyc
a
b+c ; ....
a(a + b + c)
.(b c)2 0obvious
bc(a + b)(a + c)
we have
X
1
=2
1+x
or
xy + yz + zx + 2xyz = 1
xy
1
1 + 2z
1
c
By AM-GM we have:
(x2 + m)(y 2 + m) = (xy)2 + m(x2 + y 2 ) + m2 (xy + m)2
Done! 25, Let a, b, c > 0. Prove that
X a
(a + b)2
2
c(2c + a + b)
b+c
Setting :x =
a+b
c ;y
c+a
b ;z
b+c
a
P
X x2
(x + y + z)2
x
1
P
b+c
2a ; y
c+a
2b ; z
Xa+b
c
a+b
a + b + 2c
x
x+1
a+b
2c
<=>
We have xyz 1
=> LHS
x2
X 2x
x+1
Xa+b
(a + b)2
c(2c + a + b)
2c
a+b
X
p
(a + b + 2c)c
Xr
a
2a + b + c
10
1t2
3 (t
0)
So we change
(
1 t2 (1 t)2 (1 + 2t)
,
) instead of (q, r)
3
27
<=> t2 (t 1)2 0
cyclic
q
n(a21 + a22 + ... + a2n )
cyclic
Setting a :=
becomes
ab, b :=
bc, c :=
1
3
2
ab 1
1
1
1
3
+
+
a1 b1 c1
2
P
X 1
(b 1) (c 1)
(ab + bc + ca) 2 (a + b + c) + 3
=
=
a1
(a 1) (b 1) (c 1)
abc (ab + bc + ca) + a + b + c 1
p3 + 9p2
18 + 4p
3p + 7p 11
p3 + 9p2
18 + 4p
90
X a b+c
2
b+c+1
cyc
Another solution. Applying AM-GM Inequality, we have:
a(b + c)
.
1
2
Thus, we only need to prove that:
a
2
2
b + c( a + 2)
sym
X
(a + b + c)2
(a + b + c)2
a
pP P
P
p P P
P p
b + c( a + 2)
2 a b + c + a a(b + c)
2 2( a)( ab) + ( a)( ab(a + b))
2t 2 2u t2 u2 3
By AM-GM ineq, u2 <= t2 /3. Thus,
3
2
5 6 8 3 5 8 4
2 2
( 2t 2 2u) t u + 3 t
t + t +30
3
3
3
Using AM-GM:
r
122
5 6 8 4
8 3 5
24 t
t + t + 3 24
t (t 3)
37
3
3
3
3
12
2
2
2
3a
3b
3c
2
Are you sure that this inequality is very hard. Huh?
Using AM-GM inequality we have
a3
a3
a3
a3
a2
=
= q
=p
2
2
2 3a2
3a
a(3 a )
2
a2 (3 a2 )2
2. a2 . 3a
2 . 2
35, Let a, b, c > 0 such that a + b + c + 1 = 4abc. Prove that
1
1
1
3
+
+
a4 + b + c b4 + c + a c4 + a + b
a+b+c
The inequality comes from: If a, b, c > 0 such that a + b + c 3 then
1
1
1
3
+
+
a4 + b + c b4 + c + a c4 + a + b
a+b+c
This is a very known result.
We have :
a4 + b + c =
(b2 )2
(a2 )2
(c2 )2
(a2 + b2 + c2 )2
+ 3 + 3
1
b
c
1 + b3 + c3
3 + 2(a3 + b3 + c3 )
1
a4 + b + c
(a2 + b2 + c2 )2
()
(a2 + b2 + c2 )2
a+b+c
with : a, b, c > 0 and a + b + c + 1 = 4abc a + b + c = p; ab + bc + ca = q; abc = r
We have :
4
4abc = a + b + c + 1 4 abc abc = r 1
() (p2 3q)2 + (q 2 3p) + 2(q 2 3pr) 0
We have:
(p2 3q)2 + (q 2 3p) + 2(q 2 3pr) (p2 3q)2 + 3(q 2 3pr) 0
We have done Mr. Green 36, Let a, b, c be nonnegative real numbers, no two of which
are zero. Prove that
s
s
s
8b(c + a) + 9ca
8c(a + b) + 9ab
8a(b + c) + 9bc
+
+
5
(2b + c)(b + 2c)
(2c + a)(c + 2a)
(2a + b)(a + 2b)
13
b
a
+
1
2
2
+ 3b
b + 3a2
a2
Well, well. The problem is rather good although it only contains 2 variables a, b. Interesting enough. Here is my solution. We set that
x=
a2
a
b
and y =
2
2
+ 3b
b + 3a2
+
(a(a2 +3b2 )+b(b2 +3a2 )) (a+b)3 == a(a2 +3b2 )+b(b2 +3a2 ).
a2 + 3b2
b2 + 3a2
38, Let a, b, c be positive real numbers. Prove that
3
bc + 4ab + 4ac
ca + 4bc + 4ba
ab + 4ca + 4cb
+
+
b+c
c+a
a+b
2 2
b+c
+
a
c+a
+
b
b+c
b + c (b + c)2
b+c
It suffices to prove that
3 X
b+c X
16a + b + c
b+c
X b+c
Xb+c
X (a + b)(a + c) X b + c
X a + bc
=
+2
+2
a
a
bc
a
bc
=
Xb+c
a
+2
X a
Xb+c
X a
+6
+4
+6
a
b+c
bc
14
a+b
c
X 1 1
X a
X a
a
+
+4
+68
+6
b
c
b+c
b+c
and
X
16a + b + c
b+c
!2
=
X 16a + b + c
X 16a + b + c
b+c
b+c
+
s
+2
X 16a + b + c
= 18
a+c
a
+ 54
b+c
(16a + b + c)(16b + c + a)
(a + c)(b + c)
16b + c + a
b+c
b+c
2
This is Nesbitts Inequality. Equality holds if and only if a = b = c. We have done.
1
5
a+b
2
ab
c ;y
ca
b ;z
LHS
1 1
1
5
+ +
a b a+b
2
bc
a
(ab + bc + ca)2
5
2abc
2
15
then applying p, q, r.
41, Let a, b, c be nonegative real numbers such that a2 + b2 + c2 = 1. Prove that
a
c
b
3
+
+
2
1 + ca
1 + bc
1 + ab
a
1 + bc
We have the following result:
X a
2
1 + bc
X X
a
a
1 + bc
a
(a + b)(a + c)
Actually, we have
LHSRHS =
X
X ab(a b) ca(c a)
=
(1 + bc)(a + b)(a + c)
X
X
ab(a b)
ab(a b)
ab(a b)2 (1 c2 )
=
0
(1 + bc)(a + b)(a + c)
(1 + ac)(b + a)(b + c)
(a + b)(b + c)(c + a)(1 + ac)(1 + bc)
a
(a + b)(a + c)
(a + b)(b + c)(c + a)
9
4
8
(ab + bc + ca)(a + b + c)
9
By AM - GM Inequality, it is true.
42, Let a, b, c > 0. Prove that
(a + b)
a b
+
b
c
1
+ 2
b
a
+
b
r !
b
c
Divide by b to get
a
a b
1
2 a
( + 1)( + ) + 2 ( +
b
b
c
b
b b
Let
b
)
c
a
b
1
= x, = y, and = z
b
c
b
y)
x2 + x + y + xy + (z x y)2 (x + y)2
x + xy + (z x y)2 2x y
x2 + y 2
y2 + z2
z 2 + x2
+
+
)
z+k
x+k
y+k
x2
y2
y2
z2
z2
x2
+
+
+
+
+
)
z+k z+k x+k x+k y+k y+k
4(x + y + z)2
by Cauchy-Schwarz. Now let x + y + z = a then we only need to prove
4a2 3(a + 3k)(a k)
(a 3k)2 0, which is obviously true.
using AM-GM :
k+z
x2 + y 2
+
x+y
z+k
2
44, Let a, b, c > 0. Prove that
3
ab + bc + ca a4 + b4 + c4 + .
4
a4 +
1
a2
4
and cyclic. So
3
a2 + b2 + c2 ab + bc + ca.
4
p
(a4 + b4 + 1/8 + 1/8)/4 4 a4 b4 /64
a4 + b4 + c4 +
a2 + 1 + 2b2
b2 + 1 + 2c2
c2 + 1 + 2a2
+
+
2( ab + bc + ca)
a+1
b+1
c+1
p
p
p
a2 + 1 + 2b2 b2 + 1 + 2c2 c2 + 1 + 2a2
+
+
2( a2 + b2 + b2 + c2 + c2 + a2 )
a+1
b+1
c+1
17
a2 + b2 2ab
and
b2 + 1 2b
so
a2 + 1 + 2b2 2ab + 2b = 2b(a + 1)
So then
LHS 2(a + b + c)
So we have to prove that:
a+b+c
ab +
bc +
18
9
2
9
2
9
,
2
or equivalently,
f (a) = (2c + 1)a2 + (2c2 5c 4)a +
9
0.
2
We see that f(a) is a quadratic polynomial of a with the highest coefficient is positive.
Moreover, its disciminant is
= (2c2 5c 4)2 18(2c + 1) = (2c 1)2 (c2 4c 2) 0, as0 c 3.
Therefore, f (a) 0 and our proof is completed. Equality holds if and only if a =
1
3
2 , b = 1, c = 2 .
b+c+1 2
) )9
2
But
36 8a a(7 2a)2 = (4 a)(2a 3)2 0
Since a 3, proof is complete.
49, Given a, b, c > 0 and abc = 1. Prove that
(a2 + 1)(b2 + 1)(c2 + 1)
(a + 1)(b + 1)(c + 1)
r
3
(a + b)(b + c)(c + a)
8
now and future, I will try to solve problem,i dont creat more ineq.Thank you to all
people.
r
3
x2 + 1
3 x + 1
x+1
2
(x 1)4 (x2 + x + 1) 0
and
(a2 + bc)(1 +
bc
) (a + b)2
c
so
(a2 + bc)(b2 + ca)(c2 + ab) abc(a + b)(b + c)(c + a)
(a3 + 1)(b3 + 1)(c3 + 1) (a + b)(b + c)(c + a)
The stronger is trues and very easy:
(1 + a)3 (1 + b)3 (1 + c)3 64(a + b)(b + c)(c + a)
Y
(1 + a)16 =
!8
Y
(1 + a)(1 + b)
=
!8
Y
(1 + ab + a + b)
cyc
cyc
cyc
!4
Y
(1 + ab)(a + b)
88
cyc
Y
= 88 (a + b)4
cyc
!2
Y
(1 + ab)(1 + ac)
=
cyc
!2
= 88
Y
(a + b)4
Y
(1 + a + a(b + c))
cyc
cyc
810
Y
Y
(a + b)5 (1 + a),
cyc
(1 + a)2 (1 + b)2
4
Because of:
(1 + ab)(1 + bc)(1 + ca) = (1 + a)(1 + b)(1 + c)
20
cyc
a2 + b2
b + c2
c + a2
2(ab + bc + ca)
We have :
b2
1
4
1
+ 2
2
2
2
+c
c +a
2c + (a + b)2 2ab
1
4
1
4
5
+
)
1 2x 2c2 2x + 1
2
f (x) = 80 < 0
0
0 1
f (x) f ( ) = 24c2 20c + 10 > 0
4
We have :
1
f (x) f ( ) = c((2c 1)2 + 4) 0
4
We have done
Assume : c = min{a; b; c} We have :
a2 + b2 x2 + y 2
b2 + c2 y 2
c2 + a2 x2
ab + bc + ca xy
With :
c
c
x = a + ;y = b +
2
2
x2 + y 2
xy
2
By AM-GM we have :
x2
xy
xy
3(x2 + y 2 )
5
x2 + y 2
x2 + y 2
=( 2
)+
+
+
2
2
+y
xy
x +y
4xy
4xy
2
21
We have done.
51, Given a, b, c 0. Prove that
(a3 b3 + b3 c3 + c3 a3 )[(a + b)(b + c)(c + a) 8abc] abc(a b)2 (b c)2 (c a)2
[c(ab)2 +b(ac)2 +a(bc)2 ][
1 1 1
ab bc ca 2
(a b)2 (b c)2 (c a)2
+
+
]
(
+
+
)
=
c3 b3 a3
c
a
b
a2 b2 c2
and we get
(a3 b3 + b3 c3 + c3 a3 )[(a + b)(b + c)(c + a) 8abc] abc(a b)2 (b c)2 (c a)2
52,
(a3 b3 + b3 c3 + c3 a3 )[(a + b)(b + c)(c + a) 8abc] kabc(a b)2 (b c)2 (c a)2
kmax = 9.
Replacing a, b, c by
1 1 1
a, b, c
1
1
1
9
+ 2
+ 2
+ ab + bc b + bc + ca c + ca + ab
(a + b + c)2
X
(
a2 + ab + bc)(
cyc
1
1
1
+
+
) (1 + 1 + 1)2 = 9
a2 + ab + bc b2 + bc + ca c2 + ca + ab
22
a2
1
1
1
9
+ 2
+ 2
+ ab + bc b + bc + ca c + ca + ab
(a + b + c)2
ka2 + ab + bc
k+2
for any k 0.
55, Given a, b, c 0 and 3(ab + bc + ca) + 2abc = 27. Prove that:
a+b+c
9
2
ab + bc + ca
27
4
9
2
3(a + b + c) 2(ab + bc + ca)
ab +
bc +
ca
9
2
also nice and easy.Try it.Sorry for my bad english. But 3(a + b + c) 2(ab + bc + ca),
we can use schur.
ab +
bc +
ca
cyc
1 1 1
(a b)2
(b c)2
(c a)2
+ +
+
+
b
c a
b
c
a
2
ab bc ca
+
+
b
c
a
p
(a+2b)(b+2c) = ab+bc+bc+b2 +b2 +bc+bc+ca+ca = b(a+2c)+b(c+2b)+c(b+2a) 3 3 b2 c(a + 2c)(c + 2b)(b
and similar,we have
(a + 2b)2 (b + 2c)2 (c + 2a)2 27abc(a + 2c)(b + 2a)(c + 2b)
59, Its weaker than the well-known:
(a + 2b)2 (b + 2c)2 (c + 2a)2 27(ab + bc + ca)3
60, Given a, b, c > 0. Prove that:
(a + b)(b + c)(c + a)
(a2 + b2 )(b2 + c2 )(c2 + a2 )
8abc
(a2 + bc)(b2 + ca)(c2 + ab)
(a2 + b2 )(b2 + c2 )(c2 + a2 ) (a2 + bc)(b2 + ca)(c2 + ab)
= a2 b2 (a2 +b2 )+b2 c2 (b2 +c2 )+c2 a2 (c2 +a2 )(a3 b3 +b3 c3 +c3 a3 )abc(a3 +b3 +c3 )
a2 b2 (a2 + b2 ) + b2 c2 (b2 + c2 ) + c2 a2 (c2 + a2 ) (a3 b3 + b3 c3 + c3 a3 ) 3a2 b2 c2
P 2
c (a b)2 (a + b)2
a2 b2 (a b)2 + b2 c2 (b c)2 + c2 a2 (c a)2
+
=
2
2
I think this way can prove this problem.
p
(a2 +bc)(b+c) = a2 b+a2 c+b2 c+c2 b = c(a2 +b2 )+b(a2 +c2 ) 2 bc(a2 + b2 )(c2 + a2 ) . . .
Pn
61, ai > 0, i=1 ai = 1. Prove:
n
X
i=1
If f (x) =
x
1x
a
i
1 ai
then, f 00 (x) = 4x
4
(1x)5
n
n1
X
X
1
ai
1 ai
1 ai
1 ai
X
n2
twitht =
1 ai
P
1 ai
2
24
ai = 1, yields
X
a
1
P
i )2
(
1 ai 2
1 ai
n
n1
is trivial.
(a + b c)
c2
+ (b + a)
(b + c a)
a2
+ (b + c)
(c + a b)
b + (c + a)
3
5
(1 2a)
a2 + (1 a)
X (3a 1)(3a 2)
1
0
0
2
5
a2 + (1 a)
X
3a 2
1
(3a + 3b + 3c 3)(
2) 0
2
3
a + (1 a)
(a + b + c = 1).
63, Let a, b, c are real numbers. Prove that
(a + b)4 + (b + c)4 + (c + a)4
4
(a + b + c)4
7
16
4
(a + b + c)4 (a + b + c)4
27
7
64, Given the reals a, b, c. Prove that:
X
(a + b)
4 4
[a + b4 + c4 + (a + b + c)4 ]
7
b
ab + bc + ca
25
P
2
X a2
X (a b)2
a2 + b2 + c2
(a b)
( 2a + b) (a+b+c)(
1)
(a + b + c)(
)
b
ab + bc + ca
b
2(ab + bc + ca)
X
a+b+c
2 1
(a b) (
)0
b 2(ab + bc + ca)
use SOS method we have.
After expand, this ineq is only:
x2 + y 2 + z 2 xy + yz + zx
with x = ab2 ; ...
66, Let a, b, c be real numbers. Prove that
(b + c)4 + (c + a)4 + (a + b)4
4 4
(a + b4 + c4 + (a + b + c)4 )
7
b2
c2
a2
+
+ 2 + 3 abc 6
b
c
a
With this problem, we only use cauchy-schwarz, AM-GM.
After use Cauchy-Schwarts and Am-Gm, we only obtain an old (and easy) result:
a2 + b2 + c2
8abc
+
2
ab + bc + ca (a + b)(b + c)(c + a)
The similar and harder problem: (with the same condition)
Prove that:
r
a2
b2
c2
3( +
+ ) + ( 2 1). abc 2 + 2
b
c
a
My full proof: (it seems not use Cauchy-Schwarts Embarassed )
Using a well-known result:
a2
b2
c2
(a + b + c)(a2 + b2 + c2 )
+
+
b
c
a
ab + bc + ca
(we can prove it easily by Am-Gm)
So,
a2
b2
c2
3(a2 + b2 + c2 )
+
+
b
c
a
ab + bc + ca
Besides, we have:
8abc
abc
(a + b)(b + c)(c + a)
26
64
abc(a + b + c)3
27
c2
a2
b2
3a2 b2 c2
Its an old problem.
Its equivalent to:
X
X
abc(
a3 + 3abc
ab(a + b)) 0 (obviously trues)
if abc(a b)(b c)(c a) 0. Prove that
(b c)3
(c a)3
(a b)3
3abc
+
+
3
3
3
a
b
c
(a b)(b c)(c a)
if abc(a b)(b c)(c a) 0. Prove that:
(b c)3
(c a)3
(a b)3
3abc
+
+
a3
b3
c3
(a b)(b c)(c a)
And in fact, I proved the stronger:
(b c)2
(c a)2
(a b)2 (b c)2 (c a)2
(a b)2
+
+
c2
a2
b2
a2 b2 c2
(indeed, this stronger is also my old result)
Here is my full hint:
Let x = ab(a b); ... The first ineq becomes: x2 + y 2 + z 2 (x + y + z)2
X
X
<=> 2(xy + yz + zx) 0, <=> 2abc(
a3 + 3abc
ab(a + b)) 0
PS: Besides, I think this ineq is only trues with a, b, c 0
But, setting x =
bc
a ,y
ca
b ,z
ca
b .
2
2
2
a
b
c
a2 b2 c2
27
2
2
2
a
b
c
a2 b2 c2
We have xy + yz + zx k(x + y + z)2 , k =
1
3
x2 + y 2 + z 2 2(xy + yz + zx) x2 y 2 z 2
r2
2(x + y + z)2
=
3
3
So in inequalities
(c a)2
(a b)2
(a b)2 (b c)2 (c a)2
(b c)2
+
+
2
2
2
a
b
c
ka2 b2 c2
and k =
1
3
bc ca ab 2
27
+
+
) >
a
b
c
17
3
+
(a + b)2
(b + c)2
(c + a)2
a2 + b2 + c2
We can write it into:
X (a b)2
(a c)2
(a + b)2
a2 + b2 + c2
(a c)2
4(a c)2
(a c)2
+
P
RHS
2
2
2
(a + b) + (b + c)
(a + c)
(a + b)2
(By AM-GM).
71, Give a, b, c > 0 and ab + bc + ca = 3. Prove that:
1
p
1 + (a +
b)3
1
1
3
+p
+p
3
3
2
1
1 + (b + c) 1
1 + (a + c) 1
x2 2
)
2
28
(a + b)2
(b + c)2
(c + a)2
4
or
1
1
9
1
+
+
2
2
2
(a + b)
(b + c)
(c + a)
4(ab + bc + ca)
a
b
c
a+b+c
+
+
c+a
b+c
a+b
2
By Horder we have :
X a
X
(
)2 .(
a(b + c)) (a + b + c)3
b+c
We have to prove that :
a + b + c ab + bc + ca
With a, b, c be positive numbers such that: ab + bc + ca + abc = 4 It is VMO 1996.
We have done 73, Given a, b, c 0. Prove that:
b2
c2
ab + bc + ca
a2
+
+
+ 2
2
2
2
2
2
2
2
a + ab + b
b + bc + c
c + ca + a
a + b2 + c2
P
ab + ac + bc X
(ab + ac + bc) (a5 c a3 c2 b)
a2
Q
2 2
= 2
.
a + b2 + c2
a2 + ab + c2
(a + b2 + c2 ) (a2 + ab + b2 )
cyc
74, Given a, b, c 0 and a + b + c = 3. Prove that
a2 b + b2 c + c2 a 4
Let x = a/3, y = b/3, z = c/3 x + y + z = 1
a2 b + b2 c + c2 a 4 x2 y + y 2 z + z 2 x
4
27
(x y)2 0
77, Let a, b, c 0 such that a + b + c = 3. Prove that:
(a2 b + b2 c + c2 a) + 2(ab2 + bc2 + ca2 ) + 3abc 12
I use this ineq :
a, b, c 0; a + b + c = 3
a2 b + b2 c + c2 a + abc 4
Make the similar ineq and we have done .
Maybe this ineq can be stronger .
78, Let a, b, c be positive real numbers such that a + b + c = abc. Prove that:
p
p
p
p
(1 + a2 )(1 + b2 )+ (1 + b2 )(1 + c2 )+ (1 + c2 )(1 + a2 ) 4+ (1 + a2 )(1 + b2 )(1 + c2 )
Setting :a =
1
x2 ; b
1
y2 ; c
1
z2
By Am-Gm we have :
r
3
ab + bc + ca
28
a2 + b2 + c2
(ab + bc + ca)2
a+b+c
r
3
30
ab + bc + ca
28
a2 + b2 + c2
(a + b + c)2
+
3(ab + bc + ca)
1
(a + b + c)6
27
2(a + b + c)
ab + bc + ca
=2
q
2
2
2
a +b +c
1
(a + b + c)6
3 6 27
+ 26
(ab + bc + ca)2
3(ab + bc + ca)
It is true because
9(a + b + c)2
(a + b + c)2
26(a + b + c)2
(a + b + c)2
3(a + b + c)4
=
+
+26
(ab + bc + ca)2
ab + bc + ca
3(ab + bc + ca) 3(ab + bc + ca)
3(ab + bc + ca)
We have done.
80, Let a, b, c 0. Prove that
a
b
c
3
+
+
1
1
1
, b := , c := , the inequality becomes
a
b
c
X
b2
3
2
2
c(2a + b )
a+b+c
a5 +
ab4 + 2
a3 b2 + 2
31
a2 b3 2
a4 b + 4
a2 b2 c
X
X
X
ab4 +
a2 b3 =
(ab4 + c2 a3 ) 2
a2 b2 c
X
X
X
X
X
a3 b2 +
a2 b3 =
a3 (b2 + c2 ) 2
a3 bc 2
a2 b2 c
Adding up these inequalities, we get the result.
81, Let a, b, c > 0. Prove that
1 1 1
3a
3b
3c
+ + 2
+
+
a b
c
a + 2bc b2 + 2ca c2 + 2ab
the inequality is equivalent to :
X
(a b)2 (
3
27a2 b2 c2 + 2abc + (ab + bc + ca) 12abc (ab + bc + ca) 0
1
1
1
, b := , c := , the inequality is equivalent to
a
b
c
32
a 3abc
X
2a2
X a(a2 bc)
2a2 + bc
1
+ bc
X
a3
a
2a2 + bc
By the Cauchy Schwarz Inequality, we get
P
X
( a2 )2
a3
P 3
2a2 + bc
2 a + 3abc
3
P
6( a2 )2
6a2
P
We need to prove :
X
X
X
6(
a2 )2 3
(b + c)a3 + 4(
a2 )2
X
X
<=> 2(
a2 )2 3
(b + c)a3
X
<=>
(a b)2 (a2 ab + b2 ) 0
83, Let a, b, c 0. Prove that
(a + b + c)3
+
abc
r
3
ab + bc + ca
28
a2 + b2 + c2
33
by AM-GM we have :
r
3
a2 + b2 + c2
a2 + b2 + c2
2
+
ab + ac + bc
3(ab + ac + bc) 3
=
so :
r
3
3(a + b + c)2
ab + ac + bc
ab + ac + bc
ab + ac + bc
2
2
2
a +b +c
3(a + b + c)2
abc
ab + ac + bc
so we have to prove :
(ab + ac + bc)2 + 27 84(ab + ac + bc)
wich is true because ab + ac + bc 13 .
WLOG assume a + b + c = 3, put t = ab + bc + ca => a2 + b2 + c2 = 9 2t
LSH =
27
t
27
t
27
3
+ p
+
3
a2 b2 c2 (by AM-GM)
3
abc
abc
3
abc
t.t.(9 2t)
1
2 1
5
2
1
3
(3 a2 b2 c2 + 2
) + (27 )
+ 27 = 28
3
abc
3 abc
3
3
(by AM-GM) =>q.e.d!
*A nother result,same solution:
r
(a + b + c)3
ab + bc + ca
+
28 while a, b, c > 0
abc
a2 + b2 + c2
=
*Some general problems: Let a,b,c>0. Find the best constant of k for inequalitis:
(a + b + c)3
ab + bc + ca k
+( 2
) 28
abc
a + b2 + c2
r
(a + b + c)3
ab + bc + ca
+k
2/
27 + k
abc
a2 + b2 + c2
1/
34
3
3
3
3
3
3
a + (a + b)
b + (b + c)
c + (c + a)
3
Let a + b + c = 3, we get a < 3, note that
a3
a3
1 4
1 (a 3)(a 1)2
(a 1) =
0
3
+ (3 a)
9 9
3 a2 3a + 3
Another solution :
a2
a3
(
)2
a3 + (b + c)3
a2 + b2 + c2
X
a3
a
1
3. = 1
+2
3
ab + bc + ca
ab + bc + ca
2
,
2
2
2
a +b +c
a + b2 + c2
and:
abc
(ab + bc + ca)2
,
3(a + b + c)
then
ab + bc + ca
ab + bc + ca
2
a2 + b2 + c2
a + b2 + c2
So, we only need to prove that
(a + b + c)3
ab + bc + ca
28
+ 2
abc
a + b2 + c2
To prove this inequality, we can use the known (ab + bc + ca)2 3abc(a + b + c). and
so we only need to prove
ab + bc + ca
3(a + b + c)4
+ 2
28
2
(ab + bc + ca)
a + b2 + c2
To prove this inequality, we just setting x =
3
ab+bc+ca
a2 +b2 +c2
1, then it becomes
2
1
+ 2 + x 28
x
3
t = ab + bc + ca 3 a2 b2 c2 => a2 + b2 + c2 = 9 2t
s
r
27
t3
t3
27
27
26
1
+
+ abc =
+
+ abc 28
LSH =
abc
t.t.(9 2t)
abc
27
abc
abc abc
87, Let a, b, c > 0. Prove that
a2
b2
c2
+
+
b
c
a
a3 + b3
+
a+b
b3 + c3
+
b+c
c3 + a3
c+a
15(a2 + b2 + c2 ) 3
(a + b + c)
2(a + b + c)
2
2
X
X
15(a2 + b2 + c2 ) 3
(a + b + c) 3(2
a2
ab)
2(a + b + c)
2
15
3
(1 2x) )2 3(2(1 2x) x)
2
2
(6 15x)2 3(2 5x)
36 180x + 225x2 6 15x
30 165x + 225x2 0
10 55x + 75x2 0
2 11x + 15x2 0
(1 3x)(2 5x) 0
2
+
0
0.
4a
3 4a
9
4a
cyc
cyc
cyc
89, Let a, b, c 0. Prove that
a3
b3
c3
1
a3
+ 3
+ 3
3
3
+ (a + b)
b + (b + c)
c + (c + a)3
3
1
1
with klm = 1
3
1 + (1 + k)
3
yz
x2
6
2
3
x + (x + yz)
3
x6
(x3 + y 3 + z 3 )2
P 6
2
3
+ (x + yz)
x + (x2 + yz)3
x + (x2 + yz)3
3
37
which is equivalent to
x6 + y 6 + z 6 + 5(x3 y 3 + y 3 z 3 + z 3 x3 ) 3xyz(x3 + y 3 + z 3 ) + 9x2 y 2 z 2
By AM-GM we have
x6 + x3 y 3 + x3 z 3 3x4 yz
and similar for the others and also by AM-GM
3(x3 y 3 + y 3 z 3 + z 3 x3 ) 9x2 y 2 z 2
90, Let a, b, c 0 such that a + b + c = 3. Prove that
a2
b2
c2
9
+
+
p
b+c c+a a+b
2 3(ab + bc + ca)
Of cause!
a2
b2
c2
9
+
+
p
b+c
4
2
2 3(ab + bc + ca)
cyc
p
2(a + b + c) a + b + c 3(ab + ac + bc)
X (2a b c)(2a + b + c)
p
b+c
3(ab + ac + bc)
cyc
X (a b)(2a + b + c) (c a)(2a + b + c)
b+c
b+c
cyc
P
2(a + b + c) (a b)2
p
b+c
a+c
cyc
P
X
2(a + b + c) (a b)2
p
(a b)2 Sc 0,
3(ab + ac + bc) + (a + b + c) 3(ab + ac + bc)
cyc
Sc =
1
1
p
.
Let
a b c.T henSb 0, Sc 0and(a c)2 (b c)2 .
Thus,
X
cyc
38
a + b + 2c
2
p
, which obviously.
(a + b)(a + c)(b + c)
3(ab + ac + bc) + (a + b + c) 3(ab + ac + bc)
2
b
a
Hence, it suffices to prove that
X1 X
1
2
a
a ab + b2
By the Cauchy Schwarz Inequality, we have
X
2
a ab + b2
2
X
1
ab
X
ab
2
a ab + b2
By AM-GM, we have
ab
3
ab + b2
2
X 1
1 X1
ab
3
a
X
a2
2
a ab + b2
s
X
3
1
2
a ab + b2
(a + b + c)
(ab + bc + ca)
s
X
3
39
1
2
a ab + b2
a2 ab + b2
b2 bc + c2 = 7q 2 5q 2r + 1
q 2 (1 q)
2 (2 3q)
we have
f (r) f
q 2 (1 q)
2 (2 3q)
=
2 (2 3q)
1
27
a2
b+c
X
a2
b+c
X
X
a2
b+c
2
X
a2
b+c
2
4q9
3
then we need to
(q 3) (q 0, 0885) (q 5, 487) 0
which is true so we have done. The equality holds if a = b = c.
93, Let a, b, c 0 and ab + bc + ca = 3. Prove that
r
a2 b2 + b2 c2 + c2 a2
a + b + c abc + 2
3
The first I think you have a mistake. Setting p = a + b + c and r = abc then the
inequality becomes
r
9 2pr
pr
3
3p2 4pr + 3r2 9 0
0 = 4p2 3 3p2 9 = 27 5p2 0
And the equality does not hold.
94, Let a, b, c be positive real numbers such that abc = 1. Prove that
1
1
1
2
+
+
+
1
2
2
2
(1 + a)
(1 + b)
(1 + c)
(1 + a)(1 + b)(1 + c)
we use the fact that if a, b 1 or a, b 1 hence we have :(a 1)(b 1) 0 this mean
that : ab + 1 a + b hence:
2
2
c
=
(1 + a)(1 + b)(1 + c)
(1 + c)(1 + ab + a + b)
(1 + c)2
but we also have :
1
1
ab(a b)2 + (1 + ab)2
1
c
+
=
+
(a + 1)2
(1 + b)2
(ab + 1)(a + 1)2 (b + 1)2
ab + 1
c+1
41
thus we have :
LHS
c
c
1
+
= 1 = RHS
+
2
c + 1 (c + 1)
(c + 1)2
Setting :
xy =
1
1
1
; yz =
; zx =
(1 + a)2
(1 + b)2
(1 + c)2
1
1
1
3
+ 2
+ 2
+1 b +1 c +1
2
p(12 p2 )
9
q 2 2pr + 2p2 4q + 3
(q 1)2 + (p r)2
3
2p2 2pr
3
q 2 2pr + 2p2 4q + 3
2
2
2
(q 1) + (p r)
2
4 + (p r)
2
4p2 4pr 12 + 3(p r)2 = 12 + 3p2 6pr + 3r2
p2 + 2pr 12 + 3r2
from Am-GM ;
(a + b + c)(ab + bc + ca) 9abc p 3r
pr 3r2
from Am-Gm ;
p2 q + 3pr 4q 2 p2 + pr 12
42
p2 + 2pr 12 + 3r2
3
2
96, If x, y, z are non-negative numbers such that xy + yz + zx = 3 then
LHS
x2 + y 2 + z 2 + 3xyz 6.
Note that from
xy + yz + zx = 3 => x + y + z 3
then
3xyz
9xyz
(x + y + z)
9xyz
6 = 2(xy + yz + zx)
(x + y + z)
a+b+c 5
a2 + b2 + c2
)
3
3
3abc(a + b + c)(a2 + b2 + c2 )
(ab + bc + ca)2 (a2 + b2 + c2 )
(a+b+c)5 /27
a+b+c
a+b+c
100, Let a, b, c, d > 0; 4 a + b + c + d. Prove that:
1
1
1
1
+
+
+
1.
(a + 1)2
(b + 1)2
(c + 1)2
(d + 1)2
43
LHS
101, Let a, b, c > 0. Prove that:
cyc
a2 c + b2 a + c2 b a2 b b2 c c2 a = (b a)(c a)(c b)
102, For a, b, c real numbers such that a + b + c = 1. Prove that
X
1
27
1 + a2
10
I have seen the solution use targent line .But I can prove it by cauchy-schwarz Mr. Green
<=>
X
a2
3
a2
2
+ (a + b + c)
10
We have :(a + b + c)2 (|a| + |b| + |c|)2 Setting :x = |a|; y = |b|; z = |c| Assume
that :x + y + z = 1 By cauchy-schwarz ,we need to prove :
X
10(x2 + y 2 + z 2 )2 3(x2 + y 2 + z 2 ) + 3
x4
<=> 17q 2 11q 6r + 2 0
By Am-Gm we have :r
q
9
2q
2
+ 2 = (3q 1)(17q 6) 0
3
3
11 a2 + b2 + c2
11 25
a
+ 1)2
.
+
b+c
4 ab + bc + ca
4
4
a
b
c
11(a2 + b2 + c2 ) 7
+
+
+ 1)2
+
b+c c+a a+b
4(ab + bc + ca)
2
44
1 q + 2r 2
11
)
2
qr
4q
F.4q 1
W eprove :
11
(1 q)2
2
2
q
4q
(4q 1)(3q 4) 0(Rightbecauseq
1
)
4
F.4q 1
Use schur, we have :
r
4q 1
q7 2
LHS (
)
9
5q + 1
We prove that :
(
q7 2
11
)
2
5q + 1
4q
1
1
q
3
4
104, Let a, b, c are positive numbers such that a + b + c = 3. Prove that
X
a
1
2 + 2c
b
cyc
X
cyc
1
a
(a + b + c)2 2
1 <=>
b2 + 2c
ab + bc2 + ca2 + 2(ab + bc + ca)
a2 + b2 + c2 ab2 + bc2 + ca2
a3 +
a2 b 2
ab2
which is always true
F.Lemma : a + b + c = 3; a, b, c 0
45
4(ab + bc + ca) 9
)
3
Setting :x = ab + bc + ca
4(ab + bc + ca) 9
3
5 + abc 5 +
P
We have to prove: 3 ( ab). Which is obvious true.
105, Let a, b, c > 0 and a2 + b2 + c2 = 3. Prove that
X
1
1
a3 + 2
X
1
a3 + 2
cyc
1
1 a2 1
+
a3 + 2 3
6
X a2 (a + 2)(a 1)2
a3 + 2
cyc
0
0.
2m
2n
2p
;b =
;c =
.
n+p
m+p
m+n
So we must prove :
X
r
(3
2a 2
) 3
b+c
46
(a + b + c + d)2
a
P
2
1+b c
a + b + c + d + ab2 c
(a + c)(b + d)
2
2
ab + bc + cd + da
2
a+b+c+d
2
2
4
=4
A similar problem posted in the same topic, proven in a similar way as well; but the
proof isnt quite obvious at first glance: 107, Problem Let a, b, c and d be non-negative
numbers such that a + b + c + d = 4. Prove that
a2 bc + b2 cd + c2 da + d2 ab 4
Solution The left side of this inequality cannot be factorized as we did in the previous
one. But we do see that it can be written as
ac(ab + cd) + bd(ad + bc) 4
Wed be done if we could make ab + cd appear on the left instead of ad + bc. So lets
assume that ad + bc ab + cd. Then we have
2
ac + bd + ab + cd
ac(ab + cd) + bd(ad + bc) (ac + bd)(ab + cd)
=
2
(a + d)(b + c)
2
2
1
4
a+b+c+d
2
4
=4
and were done! Now it remains to deal with the case ab + cd ad + bc. But due to
the symmetry in the expression this case is easily dealt with in exactly the same way:
ac(ab + cd) + bd(ad + bc) (ac + bd)(ad + bc)
(a + b)(c + d)
2
2
47
ac + bd + ad + bc
2
a+b+c+d
2
4
=4
2
=
Thus we are done! Some harder problems: 108, Problem (ISL2004, A5) If a, b, c are
three positive real numbers such that ab + bc + ca = 1, prove that
r
r
r
1
3 1
3 1
3 1
+ 6b +
+ 6c +
+ 6a
a
b
c
abc
Solution Note that
1
7ab + bc + ca
+ 6b =
a
a
1
2
(abc) 3
r
3
X 2 X
a
9
bc
1
[3abc(a + b + c)]2 (ab + bc + ca)4
(abc)2
a2 (b c)2 0
a2 + abc
b2 + abc
c2 + abc
1
+
+
c + ab
a + bc
b + ca
2 abc
Solution Note that
a2 + abc X
=
c + ab
p
a(c + a)(a + b)
(b + c)(c + a)
48
Now
p
a(c + a)(a + b)
a+b+c
(b + c)(c + a)
2 abc
X
p
1
a(a + b) bc(c + a)(a + b) (a + b + c)(a + b)(b + c)(c + a)
2
which was what we wanted. Another one with square-roots and fractions: 110. Problem Let a, b, c > 0. Prove that
s
r
r
r
a b
c
2a
2b
2c
+
+
3
+ +
b+c
c+a
a+b
b
c a
X
=p
b+c
(a + b)(b + c)(c + a)
(a + b)(b + c)(c + a)
Therefore it remains to show that
X
X
X X
X
X
ab2
a2 b +
ab2 + 2abc
a2 + 3
bc 3
a
2abc
Let p =
a2 b, q =
a3 +6abc(p+q+2abc)+2abc(p+q+3abc) 3q(
X
2abc
a3 + 8abcp + 8abcq + 18(abc)2 3pq + 3q 2 + 6abcq
X
X
X
2abc
a3 +8abcp+2abcq+18(abc)2 3
a3 b3 +3abc
a3 +9(abc)2 +3q 2
X
X
8abcp + 2abcq + 9(abc)2 3
a3 b3 + abc
a3 + 3q 2
Now verify that
q 2 3abcp
a2 b4 abc
a2 b
b2 (ab c2 )2 0
which is obviously true. Thus q 2 3abcp and q 2 3abcq (the latter follows directly
from AM-GM), which imply 3q 2 8abcp + abcq. Therefore it remains to show that
X
X
abcq + 9(abc)2 3
a3 b3 + abc
a3
which follows from adding the following inequalities, of which the former follows from
AM-GM and the latter from Rearrangement:
X
X
3
a3 b3 9(abc)2 abc
a3 abcq
Hence we are done. In the solutions to the last few problems, one may rise the
question: why do we break up the square-roots in that specific way? For example in
49
the fourth problem one could apply AM-GM for bc and (c + a)(a + b) instead of
b(c + a)andc(a + b). Here are my thoughts on this: while trying to get a stronger
bound, its always worth it to end up with a form which is much less, as less as possible, than the upper bound of the problem (especially in these sort of cases while using
AM-GM or Cauchy-Schwarz). Hence in accordance with the majorization inequality,
we try to derive an expression where the degrees of the terms minorize as much as
possible. For example, if we used AM-GM for 4bc and (c + a)(a + b) wed get [2, 0, 0]
and [1, 1, 0] terms. But if I use it on b(c + a) and c(a + b) I get all [1, 1, 0] terms, which
in the long run could possibly be useful. The same idea goes for the other problems as
well. If you want to see more examples of this, try looking at some older blog posts and
hopefully youll find some more illustrations.
111, If a, b, c > 0. show that :
X
b2
3
a
2
+ bc + c
a+b+c
X 2
a
a
>
b2 + bc + c2
a
3
2
+ bc + c
a+b+c
a
2
2
2
2b + bc + 2c
5
X
1
b+c
113, This one is more easier and i can say it comes from dduclams :
X
c
1 X 1
2
2
7a + 6ab + 7b
10 cyc a + b
cyc
114,
a
2
2b2 + bc + 2c2
5
Its obviously trues by CS and Schur Smile.
X
X
1
b+c
115, Let a, b and c ate non-negative numbers such that ab + ac + bc 6= 0. Prove that:
4b2
a
b
c
1
+ 2
+ 2
2
2
2
+ bc + 4c
4a + ac + 4c
4a + ab + 4b
a+b+c
50
a5 +
ab(a3 + b3 ) + 2abc
a2 b2 (a + b) + abc
ab 2
a2
a2 b2 (a + b) + abc
a2
nice solution,nguoivn
my solution:
2
LHS
(a3 + b3 + c3 )
P
3
a b3 (a + b) + abc a4
(a3 + b3 + c3 ) (a + b + c) 4
a3 b3 (a + b) + abc
a4
it equivalent to:
X
X
X
X
X
a7 +
ab(a5 + b5 ) + 2abc
a2 b2 2
a3 b3 (a + b) + abc
a4
<=>
(a b)
2
a5 + b5 c5 2(a + b) abc + 4ab(a + b)(a2 + ab + b2 )
(a
1)
0
(a + b)2 + 6
10
(3 a)2 + 6 10 25
cyc
cyc
51
X (a 1)2 (5 2a)
a2 6a + 15
cyc
a2 6a + 15
2
3a 2
2
3a
2
+ 15
2
b+c 2
2
a2
b+c
2
+
+ 15
1
=
a2 6a + 15
1
8
1
= 2
+ 2
6a + 15
a + 6a + 33 a 6a + 15
a2 + 6a + 33 a2 6a + 15
10
which is equivalent to (a1)2 (3a)(a+5) 0, which is true for 2.5 < a 3. Done!
117, This is the strongest of this form
X
a
1
2
4b3 + abc + 4c3
a + b2 + c2
X
a
a2 (ka + b + c)2
=
a
4b3 + abc + 4c3
X
52
X
a
b+c
2
.
118, The following inequality is true too. Let a, b and c are non-negative numbers such
that ab + ac + bc 6= 0. Prove that
2b3
b3
c3
a3
+ 3
+ 3
1
3
3
abc + 2c
2a abc + 2c
2a abc + 2b3
LHS
(a3 + b3 + c3 )
3
a b3 abc(a3 + b3 + c3 )
a6 + abc(a3 + b3 + c3 ) 2
a3 b3
a6 + abc(a3 + b3 + c3 ) 2
a3 b3
we have :
a3 + b3 + c3 3abc
So we have to prove that:
X
a6 + 3a2 b2 c2 2
a3 b3
a4 b2 2
a3 b3
sym
8b3
a2
1
+ 11abc + 8c3
3(a + b + c)
53
and
X
cyc
1
a
b
c
(a + b + c)3
a
+ 2
+ 2
a
a+b+c
+ 2bc
ab + bc + ca
X a(a b)(a c)
a2 + 2bc
2
a + 2bc b + 2ca c + 2ab
(a + b + c)2
54
1
1
1
+
+
a+b b+c c+a
We have
X
a2
and
X
a
a+b+c
(well-known result)
+ 2bc
ab + bc + ca
( a + b + c)2
( a + b + c)2
1
P
=
b+c
a(b + c)
2(ab + bc + ca)
a+b+c
(a2 + b2 + c2 )( a + b + c)2
,
ab + bc + ca
(a + b + c)2 (ab + bc + ca)
or
1
1
1
+
+
a+b b+c c+a
a+b+c
ab + bc + ac
ab(a + b)
ab(a b)2 0
(obviously trues)
We have:
LHS
a+b+c
ab + bc + ac
1
1
1
+
+
a+b b+c c+a
a+b+c
()
ab + bc + ac
Let: p = a + b + c = 1; q = ab + bc + ac 31 ; r = abc So
() <=>
2(1 2q)(1 + q)
1
qr
q
<=> q 2q 2 4q 3 + r 0
Case 1:
q
So:
1
4
1
1
q
q 2q 2 4q 3 + r 2q( q) + 4q( q 2 ) + 0
4
16
4
55
1
1
q
3
4
We have:
r
4q 1
bychur
9
So:
9(q 2q 2 4q 3 + r) 9q + 4q 1 18q 2 4q 3 = (1 12q 2 10q)(3q 1) 0
Its true because
1
1
q
3
4
124, Let a, b, c be nonnegative real numbers, prove that
s
X
bc
9abc
1+
(a
+
b)(a
+
c)
2(a
+
b
+
c)(ab
+ bc + ca)
cyc
125, Let a, b, c 0. Prove that:
X
3 X a(b + c)
a
b+c
4
b2 + bc + c2
3 X a(b + c)
a
b+c
4
b2 + bc + c2
3 a(b + c)
a
2
2
4 b + bc + c
b+c
a
3(a3 + b3 + c3 )
1+
b+c
2(a + b + c)(a2 + b2 + c2 )
+
+
+a+b+c
2(a2 + b2 + c2 )
c+a
b+c
a+b
128, If a, b, c be sidelengths of a triangle, then
r
b
c
3(a3 + b3 + c3 ) a + b + c
a
+
+
+
a2 + b2 + c2
2
c+a
b+c
a+b
By cauchy-swarchz.We have:
LHS 2 (a + b + c)(
a
b
c
+
+
)
b+c c+a a+b
56
9abc
9
4(ab + bc + ac)
4
9
abc
9
abc
.
= .
4 ab + bc + ca
4 (ab + bc + ca)(a + b + c)
Done!
My solution Very Happy
Setting:
ab + bc + ac = x;
1
1
1
+
+
=y
2
2
(a + b)
(b + c)
(c + a)2
So:
xy
57
9
4
Because:
a + b + c = 1 => abc
x
9
Inequality
9
9
y)
4x
4
x 9
9
LHS xy + (
y)
9 4x
4
8
1
9
<=> xy +
9
4
4
Its true Very Happy. Done!
<=> xy + abc(
133, Let a, b, c be nonnegative real numbers such that max(a, b, c) 4min(a, b, c).
Prove that
2(a + b + c)(ab + bc + ca)2 9abc(a2 + b2 + c2 + ab + bc + ca)
Equality holds for a = b = c or a = 4b = 4c
Nice: Inequality
<=> Sa (b c)2 + Sb (c a)2 + Sc (a b)2 0
With:
Sa =
* a b c => a 4c
=> Sa 0; Sb 0; Sa + Sc =
a
b+c
2
+
b
c+a
2
+
c
a+b
2
3(a2 + b2 + c2 )
4(ab + bc + ca)
a 2
b 2
c 2
) +(
) +(
)
b+c
c+a
a+b
1 2
1 2
1 2
1 2
(a + b2 + c2 )[(
) +(
) +(
) ]
3
b+c
c+a
a+b
58
1 2
1 2
1 2
9
) +(
) +(
) ]
b+c
c+a
a+b
4
X
cyc
X a3
a
+
4
3(a2 + b2 + c2 )
(b + c)2
b+c
By AM-GM
X
9
a
2
(b + c)
4(a + b + c)
X a4
X
+ a3 + b3 + c3 3
bc(b + c)
b+c
By AM-GM
X a4
a3 + b3 + c3
b+c
2
It suffices to show that
a3 + b3 + c3 + 3abc ab(a + b) + bc(b + c) + ca(c + a)
Its Schur!
135, Let a, b, c be nonnegative real number. Prove that
a
b+c
2
+
b
c+a
2
+
c
a+b
2
+
5(a2 + b2 + c2 )
1
2
4(ab + bc + ca)
136, Let a, b, c be nonnegative real number, no two of which are zero. Prove that
(a)
(b)
+
+
1
2
2
a + b + 1 b + c + 1 c + a2 + 1
59
Because
a2 + 1 = a2 + bc + ab + ca 2a bc + ab + ca = a( b + c)2
hence
X
a
a
=1
a + b2 + 1
a + a( b + c)
2
(a + b)
4(a + b + c)
The first Inequality is Iran TST 1996 Inequality (af tersettingx = ab, y = bc, z =
ca).
139, Prove that for all a, b, c be nonnegative real numbers, we have
b2
c2
2 (a + b + c)2
a2
b2
c2
a2
+
+
+
+
b+c c+a a+b
3 ab + bc + ca 2a + b + c 2b + c + a 2c + a + b
140, For all a, b, c be nonnegative real numbers. Prove that
a4
b4
c4
ab + bc + ca
a3
b3
c3
+
+
+
+
+
2
2
2
(b + c)
(c + a)
(a + b)
4
b+c c+a a+b
141, Let a, b, c be sidelengths of a triangle. Prove that
X
1
3 2
1
1
2
2
(a + b + c )
+
a2 + bc
4
a3 b + b3 c + c3 a ab3 + bc3 + ca3
cyc
Very nice inequality, Dduclam. My solution proved that
X
12(a2 + b2 + c2 )
3
1
P
P 2 2 (a2 +b2 +c2 )
2
2
2
a + bc
3 ab(a + b ) + 2 a b
4
1
1
+ 3
3
3
3
a b + b c + c a ab + bc3 + ca3
But I think it is not nice. I think you have a better proof, may I see it, my friend
142, If a, b, c be sidelengths of a triangle. Prove that
1
1
1
2(a + b + c)2
1
1
1
+
+
+
+
a2 + bc b2 + ca c2 + ab
3(a2 + b2 + c2 ) (a + b)2
(b + c)2
(c + a)2
143, Let a, b, c be nonnegative real numbers. Prove that
a(a + b)(a + c) b(b + c)(b + a) c(c + a)(c + b)
(a + b + c)4
+
+
(b + c)3
(c + a)3
(a + b)3
6(ab + bc + ca)2
144, Let a, b, c be nonnegative real numbers. Prove that
X
cyc
4
1
(b + c)(b2 + bc + c2 )
(a + b)(b + c)(c + a)
60
.
from am-gm
(a + b + c)(ab + bc + ca) 9abc, soab + bc + ca 3abc
(a + b + c)2
a2 + b2 + c2 + 6abc
2
=1
2
2
+ b + c + 6abc
a + b2 + c2 + 6abc
146, Let a, b, c be non-negative real numbers such that a + b + c = 3. Prove that
a2
a
b
c
+
+
1
2a + bc 2b + ca 2c + ab
X
X
2a
a
1
2
2a + bc
2a + bc
cyclic
cyclic
(1
cyclic
2a
)1
2a + bc
by Cauchy-Schwarz ;
X
cyclic
X
(ab + bc + ca)2
bc
(bc)2
=
2a + bc
2abc + b2 c2
6abc + a2 b2 + b2 c2 + c2 a2
cyclic
a2 b2 + b2 c2 + c2 a2 + 2abc(a + b + c)
=1
6abc + a2 b2 + b2 c2 + c2 a2
147, Let a, b, c be sidelengths of a triangle which perimeter 3. Prove that
=
1
1
1
9
+
+
3
3
ab + bc + ca
b+ca
c+ab
a+bc
1
1
1
9
+
+
4
4
ab
+
bc
+ ca
b+ca
c+ab
a+bc
1 + a2
1 + b2
1 + c2
2
The expanding gives:
X
cyc
1
3
(a b)2 (a c)2 (b c)2 + 4abc(a3 + b3 + c3 3abc) 0.
1 + a2
2
61
1
a2 +1
= 2. Prove:
ab + bc + ac
3
2
simple solution:
1
1
1
a2
b2
c2
+
+
=2
+
+
=1
2
2
2
2
2
1+a
1+b
1+c
1+a
1+b
1 + c2
by cauchy:
(a2 +1+b2 +1+c2 +1)(
b2
c2
a2
3
+
+
) (a+b+c)2 (ab+bc+ca)
2
2
1+a
1+b
1 + c2
2
2
1
(ab+bc+ca)3 + (ab+bc+ca)2 = 1
3
3
+
+
2
2
2
2c
2a
2b
b+c c+a a+b
152, for all a, b, c 0 we have following Inequality
a
b
c
2 2
1
1
1
2
2
+
+
(a + b + c )
+
+
b+c c+a a+b
3
(a + b)2
(b + c)2
(c + a)2
153, Let a, b, c 0. Prove the following inequality:
a
b
c
2
1
1
1
+
+
(ab + bc + ca)
+
+
b+c c+a a+b
3
(a + b)2
(b + c)2
(c + a)2
X 3a
X 2ab
2c
[
+
]
b+c
(a + b)2
a+b
X a
X 2ab
b+c
(a + b)2
X ab + ac 2bc
0
(b + c)2
a b c; ab + ac 2bc bc + ba 2ca ca + cb 2ab
1
1
1
(b + c)2
(c + a)2
(a + b)2
62
Applying Chebyshev Inequality. Done 154, Let a, b, c 0. Prove the following inequality:
(
a
1
1
b
c 2
1
+
+
]
+
+
) (ab + bc + ca)[
b+c c+a a+b
(a + b)2
(b + c)2
(c + a)2
3
ab(ab + 1)
ab + 1
ab + bc + ca + 3
156, Prove that for all a, b, c be nonnegative real numbers, we have
a
b
c
a3 +b3 +c3 +6abc a2 (b+c)+b2 (c+a)+c2 (a+b)+2abc
+
+
b+c c+a a+b
Its equivalent to
(a b)2 (a c)2 (b c)2 +
cyc
which is obviously true. 157, Prove that for all a, b, c be nonnegative real numbers, we
have
c+ab
a+bc
1
b+ca
+ 2
+ 2
2
5a + 4bc 5b + 4ca 5c + 4ab
a+b+c
158, For a, b, c be nonnegative real numbers, we have
2
1
1
1
2
+
+
+
2
2
2
2
2
2
ab + bc + ca
a + bc
b + ca
c + ab
a +b +c
159,
X 2
1
a+b
a2 + bc
cyc
cyc
For along time, we havent got solution for this problem yet. Now, I will post my solution
Problem (Vo Quoc Ba Can) Let a,b,c be nonnegative real numbers. Prove that
X
X 2
1
.
a+b
a2 + bc
cyc
cyc
Solution.
By the Cauchy schwarz Inequality, we have
X
2 X
X
1
1
(a + b)(a + c)
(a + b)(a + c)
a2 + bc
a2 + bc
63
P
X
2 a
a(b + c)
+
3
(a + b)(b + c)(c + a)
a2 + bc
+
3
a2 + bc
(a + b)(b + c)(c + a) a
P 4 P 2 2
X
X a(b + c)
a a b
1
1
P
3
(ab)(ac)
+
0
a2 + bc
(a + b)(b + c)(c + a) a
a2 + bc (b + c)(a + b + c)
Due to symmetry, we may assume a b c, since a c ab (b c). It suffices to
show that
1
1
1
1
a
+
b
+
a2 + bc (b + c)(a + b + c)
b2 + ca (a + c)(a + b + c)
c(a2 b2 )[(a b)2 + ab + bc + ca] 0
which is trivial. Equality holds if and only if a = b = c.
160, Let a, b, c be non-negative real numbers. Prove that
b
c
a
b
c
a
+
+
+
+
2
2
2
b+c c+a a+b
a + 3bc
b + 3ca
c + 3ab
After using Cauchy Schwarz, we can see that the inequality follows from
X
that is
X a(b + c)
a
,
b+c
a2 + 3bc
X a(a2 + bc b2 c2 )
(b + c)(a2 + 3bc)
or
X a3 (b + c) a(b3 + c3 )
(b + c)2 (a2 + 3bc)
0,
0.
1
1
1
.
(b + c)2 (a2 + 3bc)
(c + a)2 (b2 + 3ca)
(a + b)2 (c2 + 3ab)
It follows that
a3 (b + c) a(b3 + c3 )
a3 (b + c) a(b3 + c3 )
,
(b + c)2 (a2 + 3bc)
(c + a)2 (b2 + 3ca)
64
and
c3 (a + b) c(a3 + b3 )
c3 (a + b) c(a3 + b3 )
.
2
2
(a + b) (c + 3ab)
(c + a)2 (b2 + 3ca)
Therefore
X a3 (b + c) a(b3 + c3 )
(b + c)2 (a2 + 3bc)
P 3
(a (b + c) a(b3 + c3 ))
= 0.
(c + a)2 (b2 + 3ca)
+
+
2(ab + bc + ca)
a2 + 3bc
b2 + 3ca
c2 + 3ab
161, Let equation:
(x + 1).lnx x.ln(x + 1) = 0.
Prove that this equation have only one root
I can prove it.
x0
x+1
x
x
)+
x+1
x
x+1
x
<1
x+1
1
g(x) = lnt + t
t
1
1
g 0 (x) = 2 1 0t R
t
t
=> g(x) g(1) = 0
t=
=> f 0 (x) 0
=> ...
But we have :
f (...) = ... > 0
162, For positive a, b and c such that a + b + c = 3. Prove that:
b2
c2
a2
+
+
1
2a + b2
2b + c2
2c + a2
65
P
X
a2
a4
( a2 )2
P
P
=
2a + b2
2a3 + a2 b2
2 a3 + a2 b2
X
a4
(a2 + b2 + c2 )2
a2
=
3
1
2
3
2
2
3
a + 2b
a + 2a b
(a + b + c3 ) + 2(a2 b2 + b2 c2 + c2 a2 )
a4
(a3 b + ab3 )
which is just Muirhead.Or you can also prove this last inequality by AM-GM.
163, Let a, b, c > 0 and abc = 1. Prove that
b2
c2
a
b
c
a2
+
+
+
+
1 + 2ab 1 + 2bc 1 + 2ca
ab + b + 1 bc + c + 1 ca + a + 1
a=
x
y
z
b= c=
y
z
x
X ( xz
x2 z
y )
2
y (2x + z)
x+y+z
cyc
(x + y + z)
X
cyc
X xz
x2 z
2
y (2x + z)
y
cyc
X x z
X xz
2x z
x2 z 2
( 2 +
+ 2
)2
y
y(z + 2x) y (z + 2x)
y
cyc
cyc
By Cauchy
x2 z
(2x y)z
2xz
=
z
y y
y
y
9x2
( y(2x+z)
)
z
6x
y(2x + z)
2xy
= 6x y
z
z
66
z
9x2
y(2x + z)
6xz
z
)=
2x z
(6x
y(2x+z)
y(
y
z
y
)
z
So
X 20xz
X xz
X x2 z
2x2 z
x2 z 2
2x
+ 2
)
(
)2
( 2 +
y
y(z + 2x) y (z + 2x)
9y
9
y
cyc
cyc
cyc
Let me try another solution MSetting :
x
y
z
a = ;b = ;c =
y
z
x
LHS =
RHS =
x2 y 2
z2
2xz + z 2
P xy
z
x+y+z
x2 y 2
z2
2xz +
z2
X xy
)(
)2
z
2b + 1 2c + 1 2a + 1
abc
Very nice inequality, my brother I have solved it and I hope that I have not made mistake
in solving ... Here is my solution. Note that we have a well-known inequality:
FLemma :
Let a, b, c 0 such that a + b + c = 3 we have:
a2 c + c2 b + b2 c 4 abc
It is easy to prove if we assume that
a(a b)(c b) 0.
Now we are coming back above inequality:
F Let a, b, c be positive real numbers such that a + b + c = 3. Prove that
a
b
c
1
+
+
2b + 1 2c + 1 2a + 1
abc
67
q
3
(3 q)(2q + 21)
0, 0 q 3
9
We are done
Let me try. My solution used Am-Gm .
F Lemma :
a + b + c = 3, a, b, c 0
then :
a2 b + b2 c + c2 a + abc 4 (It can prove by Am-Gm)
Expanding the ineq :
X
X
X
4(
a2 c)abc + 4abc + (
a2 )abc 4
ab + 7
X
LHS 20abc + (
a2 )abc 4a2 b2 c2
We need to prove:
X
X
20abc + (
a2 )abc 4(a2 b2 c2 + 1) + 4
ab + 3
Easy to prove :
a2 b2 c2 + 1 2abcand :
68
ab 3abc
X
a2 )[3(a + b + c)abc] (
a2 )(ab + bc + ca)2 27
X 2a2 bc
2
2b + 1
X a2 c
X
a2 c 2
2b + 1
By cauchy-schwarz , we can prove:
P
( a2 c)2
P
LHS P
2( a)abc + a2 c
<=>
2b + 1 2c + 1 2a + 1
abc
F My similar one,also nice:
1
1
1
1
+
+
2 2 2
a(2ab + 1) b(2bc + 1) c(2ca + 1)
a b c
PS: Two problems were made (stronger) from my following two ones:
F
b
c
1
1
a
+
+
3
(b + 2)2
(c + 2)2
(a + 2)2
3abc
69
1
1
1
1
1
+
2 2 2
+
2
2
2
3
a(ab + 2) ) b(bc + 2)
c(ca + 2)
3a b c
r
r
a + 2b
b + 2c
c + 2a
+
+
3; a, b, c 0
a + 2c
b + 2a
c + 2b
168, F A stronger and harder one:
r
r
r
a + 2b
b + 2c
c + 2a
+
+
3; a, b, c 0
c + 2b
a + 2c
b + 2a
169, Let a, b, c be nonnegative real numbers. Prove that
a + 2b + 3
b + 2c + 3
c + 2a + 3
+
+
3
c + 2b + 3
a + 2c + 3 b + 2a + 3
My solution assume c = min(a, b, c) and set c + 1 = z, a + 1 = z + m, b + 1 =
z + nwithm, n 0
Setting x = a + 1, y = b + 1, z = c + 1, then the inequality becomes
X x + 2y
3
z + 2y
70
x3 +
xy 2 2
x2 y
x(x y)2 0
which is true.
170, And general problems:
1,
2,
b + 2c + k
c + 2a + k
a + 2b + k
+
+
3f ork 0
c + 2b + k
a + 2c + k b + 2a + k
a + mb + n
b + mc + n
c + ma + n
+
+
3f orm, n 0
c + mb + n
a + mc + n b + ma + n
n
m+1 , y
=b+
n
m+1 , z
=c+
X x + my
z + my
n
m+1 ,
>=
>=
b+c
3abc
abc(a + b + c)
X a2
X 2a2
X
X
3
3
3
2 4
a +b +c
a a=
4
b+c
bc
71
Done
F
3
X
X 1
a2
3 3
(
a2 )(
) (a + b + c)2 p
..
b+c
b+c
2(a + b + c)
a
3
b+c 2
1
>0
(a + b)(a + c)
similar with Sb , Sc we get the same result.so the inequality has been proved.
X a
X 2a
X a
+
a3 + b3 + c3 a a + b b + c c =
b
+
c
b
+
c
2
bc
By the hypothesis, we have
a3 + b3 + c3 =
X a2
bc
4.
a 2
4 X a 2 X a
3
) .(
)
+
b+c
3
b+c
b+c 2
abc = 1 > a3 + b3 + c3 + 3
Xa+b
c
1
1
+
1
(1 + a)2
1+b+c+a
Easy expand
<=> q 2 2qp + p3 5p 3 0
0 = p2 + 3 + 5p p3 = (p 3)(1 + p)2 0
72
a
b+c
5c2 + ab
2
+ b2 ) ab
5(a2
Using Cauchy Schwarz,we have:
X
P
P
5c2 + ab
(5 a2 + ab)2
P
(1)
5(a2 + b2 ) ab
(5(a2 + b2 ) ab)(5c2 + ab)
3 (a + b)(b + c)(c + a) 2( a + b + b + c + c + a) 6 2
F First, we prove the left ineq. Setting :x = ab; y = bc; z = ca => x + y + z = 3
By cauchy-schwarz , we need to prove :
9(x + y)(y + z)(z + x) 8(x + y + z)(xy + yz + zx)(Right)
F, Next, we prove the right ineq :
By Am-Gm, we need to prove
(a + b)(b + c)(c + a) 8
<=> 3(a + b + c) abc 8(Rightbecauseab + bc + ca = 3)
177, Let a, b, c > 0 such that a2 + b2 + c2 = 3. Prove that:
(2 a)(2 b)(2 c)
25
27
f (c) = (2 c)
73
c2 + 1
2
f 0 (c) = 0 c =
1
25
1
, c = 1Hence, M inf (c) = M in f (0), f (1), f
=
3
3
27
1
3
178, Solve
log42 (x2 + 4) = 2log 12
p
8x2 + 32 + 6
log2 (x2 + 4) = a
log42 (x2 + 4) = (log4 (x2 + 4))2 =
log 12
8x2 + 32 = log2
a2
4
1
1
8x2 + 32 = (log2 8 + log2 (x2 + 4)) = (3 + a)
2
2
1
a2
= 2( (3+a))+6 = 3a+6 = a+3 > a2 +4a12 = 0 > (a+6)(a2) = 0 >
4
2
a1 = 2 > log2 (x2 + 4) = 2 > x2 + 4 = 22 > x = 0
a2 = 6 > log2 (x2 + 4) = 6 > x2 + 4 = 26 >
179, Let a, b, c > 0 such that a2 + b2 + c2 = 3. Prove that
X a
9
2(a + b + c)
b+c
its very easy for you. we have
X
X a2
a
(a + b + c)2
=
b+c
ab + ac
2(ab + bc + ac)
Xa+b
c
2
18
a+b+c
By CBS
p
(a2 + b2 )(c2 + d2 ) ac + bd
74
we have
2(1 a)(1 d) = (a + d 1)2 + b2 + c2 b2 + c2
2(1 a)(1 d) b2 + c2
2(1 b)(1 c) a2 + d2
p
(a2 + d2 )(b2 + c2 ) (ab + cd)
p
(a2 + d2 )(b2 + c2 ) (ac + bd)
181, Let triangle ABC. Prove that
(z + x) (x + y) =
z+x x+y =
b c = bc + ca + ab;
1
((y + z) + (z + x) + (x + y))
2
2
2
2
1
=
y+z +
z+x +
x+y
2
x+y+z =
75
1 2
a + b2 + c2 ;
2
1
3 yz + zx + xy = 2 3
yz + zx + xy = 2 3 S.
2
Hence, the inequality in question,
Xp
(z + x) (x + y) x + y + z + 3 yz + zx + xy,
=
becomes
1 2
a + b2 + c2 + 2 3 S.
2
Multiplication by 2 transforms this into
2 (bc + ca + ab) a2 + b2 + c2 + 4 3 S,
bc + ca + ab
or, equivalently,
2 (bc + ca + ab) a2 + b2 + c2 4 3 S.
Using the notation
Q = (b c) + (c a) + (a b)
this rewrites as
a2 + b2 + c2 Q 4 3 S,
what is equivalent to
a2 + b2 + c2 4 3 S + Q.
2 x+y+z
X
2
3(xy + xz + yz)
z + x z + y 0.
cyc
But
X
p
2
2 x + y + z 3(xy + xz + yz)
z+x z+y =
cyc
X
cyc
(x y)2
p
2
x + y + z + 3(xy + xz + yz)
z+x+ z+y
p
p
X (x y)2 z + 2 (z + x)(z + y) 3(xy + xz + yz)
=
p
2 =
3(xy + xz + yz)
z+x+ z+y
cyc x + y + z +
76
!
=
(x y)2 z +
=
X
cyc
x+y+z+
4z 2 +xy+xz+yz
(z+x)(z+y)+ 3(xy+xz+yz)
2 0.
3(xy + xz + yz)
z+x+ z+y
2
2
2
12
3+a
3+b
3+c
X
X
2xycosC
x2 > OK
P
185, Prove if a, b, c > 14 such that
a 3 then
Xp
a2 + 3 a + b + c + 3
let prove
p
1
a2 + 3 (a + 1) f(x) denotea >
4
p
1
a2 + 3 a a + 2witha.
4
p
a2 + 3 a a + 2
< > (4a 1)(a 1)2 0
186, Let a, b, c > 0 such that a+b+c=1 Prove that
1
1
X (a + b)b
cyc
c+a
X (a + b)b
c+a
cyc
X b(a + b)
c+a
<>
X a(a + b + c) + b(a + b + c a c)
<>
c+a
X a+b
a+c
cos BC
2
+2
sin B2 sin C2
X
a
pa
77
b+c
pa
because
X
Xc+b
a
pa
a
BY CBS
Let x = p a > 0 : y = p b > 0; z = p c > 0 we have
Xx+y
z
>
X
> (
x
2x + y + z
y+z
y+z
Xb+c
a
pa
a
X a
Xb+c
X
a 2
) (
)(
)(
pa
oa
a
b+c 2
)
pa
a
3
b + c3
2
By Trebusep
188, Let a, b, c > 0 such that abc = 1. Prove that
X
1
1
+
1
2
(1 + a)
1+b+c+a
Easy. Expand
<=> q 2 2qp + p3 5p 3 0
0 = p2 + 3 + 5p p3 = (p 3)(1 + p)2 0
189, Let a, b, c, x, y, z > 0. Find the minimal value of the expression
P =
By Holder
190, Prove that the sides a, b, c of any triangle suck that a2 + b2 + c2 = 3 satisfy
the inequality
X
a
1
2
a +b+c
P
a + b + c + 2ab + 2bc + 2ca
cyc a(1 + b + c)
LHS
=
1
(a + b + c)2
3 + 2ab + 2bc + 2ca
191, Prove if a, b, c > 0 then
X
p
78
b + c a = x2 , a + c b = y 2 , a + b c = z 2
We want to prove
X
x(2y 2 + z 2 + x2 )(2z 2 + y 2 + x2 ) 16xyz(x2 + y 2 + z 2 )
X
X
x(2y 2 +z 2 +x2 )(2z 2 +y 2 +x2 ) =
x((x4 +y 2 z 2 )+(2y 4 +2y 2 z 2 )+(2z 4 +2y 2 z 2 )+(3x2 y 2 +3x2 z 2 ))
X
(8x3 yz + 4y 3 zx + 4z 3 xy) = 16xyz(x2 + y 2 + z 2 )
X
p
(b + c a)(c + a b)
4(a + b + c)
(b + c a)(c + a b)
2(a + b)(b + c)
(a + b)(b + c)
=
b+ca+c+ab
c
4(a + b + c)
(1)
Since the sequences { a1 , 1b , 1c } and{(c + a)(a + b), (a + b)(b + c), (b + c)(c + a)} are
oppositely sorted, from Rearrangement we get
X (a + b)(b + c)
c
X (a + b)(b + c)
b
=a+b+c+
ca
b
a+b+c
X ca
c
=a+b+c
a+b
6
a+bc
a2
b2
c2
+
+ )2 (a2 b2 + b2 c2 + c2 a2 ) (a2 + b2 + c2 )3
b
c
a
79
and
(a2 + b2 + c2 )6
27(a4 + b4 + c4 )
+ b2 c2 + c2 a2 )2
(a2 b2
y 2 +z 2
2 ; ....
By P2 we have done
a2
b2
c2
323
+ 3
+ 3
p
b3 + c3
c + a3
a + b3
2 2 (a2 + b2 + c2
We assume a2 + b2 + c2 = 3 then the inequality becomes
b2
c2
3
a2
+
+
b3 + c3
c3 + a3
a3 + b3
2
Note that for a, b, c 0 and a + b + c = 3 then
3
a2 b + b2 c + c2 a 3
By the Cauchy Schwarz we get
2
a2 + b2 + c2
9
a2
b2
c2
3
+ 3
+ 3
P 2 3 P 3 2 =
a b + a b
b3 + c3
c + a3
a + b3
6
2
Let a2 + b2 + c2 = 3. Then we need to prove that
X
cyc
But
X
cyc
3
a2
.
3
3
b +c
2
X
a2
a4
9
.
=
P 3 2
3 a2 + c3 a2
(a
b
+ a3 c2 )
b3 + c3
b
cyc
(a3 b2 + a3 c2 ) 6.
cyc
But
(a3 b2 + a3 c2 ) 6
cyc
a3 (3 a2 ) 6
cyc
(a5 3a3 + 2) 0
cyc
cyc
a2 (a + 2)(a 1)2 0.
cyc
80
a2
b2
c2
+
+
2(a2 + b2 + c2 )
b
c
a
b
c
a
ab + bc + ca
195, SOLVE
1)x2 + x 1 = xex
+ (x2 1)ex
2)2x2 6x + 1 = log2
2x + 1
2(x 1)2
6
5
a+b+c
setting
F (a, b, c) = ab + bc + ca +
6
a+b+c
a = max(a, b, c) F (a, b, c) F (a, bc, bc)
b
a
+
1
a2 + 3b2
b2 + 3a2
81
By Holder
2
b
a
+
(a(a2 + 3b2 ) + b(b2 + 3a2 )) (a + b)3 =
a2 + 3b2
b2 + 3a2
= a(a2 + 3b2 ) + b(b2 + 3a2 ).
198, Let a, b, c > 0 such that a2 + b2 + c2 = 3. Prove that
a3 (a + b) + b3 (b + c) + c3 (c + a) 6
We have
(a + b + c)(a3 + b3 + c3 ) (a2 + b2 + c2 )2
and
3(a3 c + b3 a + c3 c) (a2 + b2 + c2 )2
199, Let a, b, c > 0. Prove that
X
2a
b+c
Xa
3(
)
b
2a
3,
b+c
a, b, cbethesidelengthsof atriangle
b+c
b
201, Let a, b, c be positive number such that a2 + b2 + c2 = 34 . Prove that
ab
bc
ca
+
+
1
ab + 1 a bc + 1 b ca + 1 c
Its easy by CS! Razz 202, Find the best positive constant k such that the following
inequalitys right
ab
bc
ca
+
+
1
ab + k(k a) bc + k(k b) ca + k(k c)
for all positive numbers a, b, c such that a2 + b2 + c2 = 1. Is k =
2 3
3
ab
bc
ca
1
+
+
1
1
1 + yzcosA + xyz 2 cosAcosB
Use:
P 2
q X
X
x
3
xyz 2 cosAcosB
yzcosA
2
and Cauchy-Schwarz inequality,we have Q.E.D
206, Let a, b, c be positive number such that
a
1+a
b
1+b
c
1+c
= 1. Prove that
2a 1
2b 1
2c 1
+
+
0
1 + 2a + 4ab 1 + 2b + 4bc 1 + 2c + 4ca
207, Let a, b, c be positive numbers. Prove that
1 1 1
(a + 2b)2
(b + 2c)2
(c + 2a)2
(a + b + c)( + + ) 2
+ 2
+ 2
a b
c
a + b(c + a) b + c(a + b) c + a(b + c)
LHS =
X
cyc
(1 +
X a2
X a + 2b2
b
b
4b2
+ )=
+
= RHS
c a
a2
bc + ba
a2 + bc + ba
cyc
cyc
+
a2 + b2
b2 + c2
c2 + a2
2
c2 + a2
Althought this ineq may be seem ugly, but the equality holds when a = b = c.
It is weaker than:
a3
b3
c3
a+b+c
+
+
a2 + b2
b2 + c2
c2 + a2
2
and an inequality very strong
a3
b3
c3
+ 2
+ 2
2
2
2
a +b
b +c
c + a2
83
3(a2 + b2 + c2 )
2
X
cyc
a2
X
X
a3
ab2
ab2
1X
=
a 2
>
a
=
a
2
2
+b
a +b
2ab
2 cyc
cyc
cyc
It remains to prove
b+c
ca2
a+b+c
>
+ 2
2
2
c + a2
which is equivalent to ac2 + a3 > 2ca2 which is true by AM-GM
209, Let x, y, z > 1. Prove that
1 + x2
1 + y2
1 + z2
+
+
2.
2
2
1+y+z
1+z+x
1 + x + y2
Ill try to clarify. As x
X
1+x2
2
we have
X
2(1 + x2 )
1 + x2
2
1+y+z
(1 + y 2 ) + 2(1 + z 2 )
X 2
a X
a
a(2b + c)
b + 2c
X 2
X
a(2b + c)
a 3(ab + bc + ca) =
p
b3
c3
a3
+ 2
+ 2
+ 2 3(a6 + b6 + c6 ) 3(a3 + b3 + c3 )
2
2
2
+c
c +a
a +b
84
Therefore:
p
p
p
(3x + 2)2 + (2x 1)2 + (5x 2)2 + (2x 1)2 |3x+2|+|5x2| |(3x+2)+(5x2)| = |8x| (8x)2
with equality occuring when
(2x 1)2 = (4x 2)2 = 0 x =
1
.
2
X cos4
X 1+
cyc
b2 +c2 a2
2bc
1 b
cos2
cyc
B
C
A
cos cos
2
2
2
2
2 +c2 a2
2bc
X
cyc
b2 + c2 a2
1+
2bc
s
a2 + c2 b2
1+
2ac
a2 + b2 c2
1+
2ab
p
X (b + c a) (a + b c)(a + c b)
(b + c a)2
bc(a + b c)(a + c b)
abc bc
cyc
cyc
p
X (b + c a) (a + b c)(a + c b)
X
(b + c a)2
bc(a + b c)(a + c b)
abc bc
cyc
cyc
s
X
X (y + z)x2
4x2 yz
2yz
(x + y)(x + z)
cyc
cyc
s
X
X
4x2 yz
3
3
(x y + x z) 2xyz
(x + y)(x + z)
cyc
cyc
X
85
(x3 y + x3 z) 2xyz(x + y + z)
cyc
and
X
cyc
1
x+y+z
s
X
cyc
X
4x2 yz
y+z
=
(x + y)(x + z)
2(x
+ y + z)
cyc
8x2 yz
=
(x + y + z)(x + y)(x + z)(y + z)
16x2 yz
8xyz
1
(x + y)(x + z)(y + z)
bc
ca
ab
a+b+c
3 3
+
+
4
a(1 + bc) b(1 + ca) c(1 + ab)
4
use the inequality:
4
1
1
+
x+y
x y
we obtain:
4bc
bc
bc
+
2a + b + c
a+b a+c
ca
ac
4ac
+
2b + a + c
a+b b+c
4ab
ab
ab
+
2c + a + b
b+c a+c
From Titus lemma, we have:
P 2
P 2
X ab
ab)
ab)
(
(
3 3abc
3 3
=
c + abc
a + b + c + 3abc
4abc
4abc
4
3
a2
4
3
3
5
a + 24b c
a2
3
4
2
2
5
a + 24b c
cyc
or equivalently:
P =
X
cyc
a
3
5
a2 + 24bc
86
!
X
a + 24abc P
cyc
cyc
so it suffices to prove
!3
X
25
!
X
cyc
a + 24abc
cyc
q
=
=
4 + 24b2 c2
4
2
2
2 2 2
a
a + 24b c
1 + 24a b c
a6
f (x) = q
1+
24a2 b2 c2
x6
f 00 (x) 0
So
f (a) + f (b) + f (c) 3f (
So the
LHS 3
a+b+c
)
3
X
q
1+
36 24a2 b2 c2
(a+b+c)6
X
q
1+
36 24a2 b2 c2
(a+b+c)6
1
(a + b + c)6 36 a2 b2 c2
5
which is true .
214, Let a, b, c be positive number such that a + b + c = 1. Find the minimum of
P =
b
c
a
+
+
2
2
2
2
1+b +c
1+c +a
1 + a2 + b2
bu Cauchy-Schwarz,
X
S =P a+b+c+
ab(a + b) (a + b + c)2 = 1
87
ab(a + b) a3 + b3 + c3 + 3
then
ab(a + b)
P
1+
1
4
1
4
S1
and finally
4
5
< a, b, c < 1 and
Pmin =
1
2
a4 + b4 + c4 + 4a2 b2 c2 = 2(a2 b2 + b2 c2 + c2 a2 )
Prove that
X
cyc
2 2
2 2
a4
4abc(a + b + c)
1 a2
2 2
a2
b2
c2
+
+
4(ab + bc + ca).
1a 1b 1c
This inequality follows from the following inequality
8(a2 + b2 + c2 ) (ab + bc + ca)
21abc(a + b + c)
2(ab + bc + ca) a2 b2 c2
X a3 + 2a
cyc
Prove that
we have
a2 + 1
9
2
X1
3
a
cyc
X a
X
9 X
=
a+
sowehave
a3
2
2
a +1
88
but
X
9 X 1 X a4 + a2 1 X
=
+
=
a+
f (a)
3
2
a
a +a
where
f (a) =
but
f 00 (a) =
a3 + a
2(6a4 + 3a2 + 1)
< 0f oreverya > 0
a3 (a2 + 1)3
so we have
X
X a4 + a2 1
f (a) 3f (
a+b+c
3
) 3f (1) =
3
2
so we have
X1
3
a
217, Let a, b and c are non-negative numbers such that ab + ac + bc = 3. Prove that:
1) a2 + b2 + c2 + 3abc 6
Using Schurs inequality
4(ab + bc + ca)(a + b + c) 9abc (a + b + c)3
Then
9abc
and
a+b+c
9abc
+ 3abc RHS
LHS = (a + b + c)2 6 + 3abc 6
a+b+c
(a + b + c)2 12
because
(a + b + c)2 3(ab + bc + ca)
2) a4 + b4 + c4 + 15abc 18
assume that: a + b + c = p, ab + bc + ca = q, abc = r so q = 3 and we have to prove
that:
p2 (12 p2 )
r
15 + 4p
case1:if p2 > 12 the ineq is true
case2: if p2 12
remember this schur ineq
r
(p2 3)(12 p2 )
6p
6p
15 + 4p
89
2
which is
equivalent to (p 3)(2p 9p 15) 0 (which is obvious true for all
p 12).
s
cos A =
bc
(b + a)(c + a)
r
Xa+b
a+b
2
+6
c
c
p
cosA + cosB + cosC 3 (cos2 A + cos2 B + cos2 C)
X
4
(cos2 A + cos2 B + cos2 C)
3
It remains to prove
cos2 A + cos2 B + cos2 C
9
4
Its obvious.....
219, Let a, b, c 0 satisfy a + b + c = 1. Prove that
(a2 + b2 )(b2 + c2 )(c2 + a2 )
1
32
let
f (a, b, c) = (a2 + b2 )(b2 + c2 )(c2 + a2 )
letc = max(a, b, c); we have
f (a, b, c) f (a + b, 0, c)
(which is equivalent)
ab(4abc2 + a3 b + ab3 4a2 c2 4b2 c2 2c4 ) 0
90
1
2
which is equivalent to
1
(16c4 32c3 + 20c2 4c 1))(1 + 2c)2 0
32
remember that
1 5
1+ 5
2
16c 32c + 20c 4c 1 = 4(2c 2c +
)(2c 2c +
)0
4
4
4
2
2bc 2ca 2ab
+
+
=
((ab)2 + (bc)2 + (ca)2 )
a
b
c
abc
But
(ab)2 + (bc)2 + (ca)2
= LHS
1
(ab + bc + ca)2
3
2
(ab + bc + ca)2
3abc
4
4abc = a + b + c + 1 4 abc = abc 1
= a + b + c 4abc 1 3abc
But
(ab + bc + ca)2 3abc(a + b + c) 9a2 b2 c2 = ab + bc + ca 3abc.
91
X a
X
4
+4
b+c
a(b c)
(b + c)
My soulution
<=>
a(b c)2 (
1
4
)0
bc(b + c (b + c)
P
(a + b + c) ( a3 (b c)2 )
abc(a + b)(b + c)(c + a)
(b + c)
(b + a)(c + a)
4(a + b + c)
bc
X a(b + c)
a
(b + c)(1 + ) = 2(a + b + c) +
4(a + b + c)
bc
bc
1a
(1 a)(1 b)(1 c)
1+b+c
92
ab + bc + ca a + b + c
3
227,
a4 + b4 + c4
3abc
(a2 + b2 + c2 )
+9
14
ab + bc + ca
a+b+c
3
But the slightly stronger is wrong:
5
3abc
(a2 + b2 + c2 )
a4 + b4 + c4
+ 10
15
ab + bc + ca
a+b+c
3
228, Problem. For all positive real numbers a,b,c we have the following inequality
r
6
6
6
a2
b2
c2
6 a + b + c
+
+
3
b
c
a
3
Use the following
X a2
b
a4 + 13
P
P
a3 (b + c) a2 b2 a2 bc
P P
3( a)( ab)
b
a+b+c
With this lenma, I have had a nice proof by Am-Gm.
229, Let a, b, c > 0. Prove that:
2
a2 + b2 + c2
(a b)2 + (b c)2 + (c a)2
a3 + b3 + c3 + abc
(
) +
4abc
ab + bc + ca
8(ab + bc + ca)
230, Let be x, y, z R+ . Show that :
p
3
3
3
2
2
2
x +
y +
z +
(x + y)(y + z)(z + x)
4
4
4
Because
3
x +
4
2
3
2
y +
x+y
4
We have
3
x + =
4
2
1
x +
4
2
1
+ 2
2
s
1
1
x2 +
2
4
Similarly we obtains
3
x +
4
2
s
3
3
1
1
1
1
2
2
y +
z +
8
x2 +
y2 +
z2 +
4
4
8
4
4
4
93
Y
4x2 + 1 8
(x + y)
x+y
y+z
z+x
3 2
p
+p
+p
2
z 2 + xy + 1
x2 + yz + 1
y 2 + zx + 1
we can let a = x + y, b = y + z, c = z + x then we have ab + bc + ca = 2 and we have
to prove that:
X
3
a
(1)
2 + 3bc
2
a
cyc
Just use Holder, let the LHS be S, then by Holder
X
S2(
a(a2 + 3bc)) (a + b + c)3
So we have to show
4(a + b + c)3 9(a3 + b3 + c3 + 9abc)
which is obvious by Muirhead. I will must prove that:
2(ab(a + b) + bc(b + c) + ca(c + a)) a3 + b3 + c3 + 9abc
<=> (3c a b)(a b)2 + (3a b c)(b c)2 + (3b a c)(c a)2 0()
Suppose a b c We have:
3b a c 3b c (b + c) = 2(b c) 0, , (c a)2 (a b)2 + (b c)2
then
LHS (3c a b + 3b a c)(a b)2 + (3b a c + 3a b c)(b c)2 0
x + y + z X x2 y 2
9
(
)
2
xyz
(x + y)
4
but:
P
x+y+z
( xy)2
LHS
P
xyz
(x + y)2
remember that:
X
(x + y)2
4
(x + y + z)2
3
94
LHS
P
P
x + y + z 3 ( xy)2
3
( xy)2
9
P 2 =
xyz
4 ( x)
4 xyz(x + y + z)
4
(a + b + c)(a2 + b2 + c2 ) 9abc
0
2abc(a2 + b2 + c2 )
ab
5
abc
X
X
X
(
a + 2)2 2(
ab) 5(
a + 2)
X
X
a2
a+6
abc
remember that:
X
abc = a + b + c + 2then
a6
X
X
(a2 + 4)
4a
X
3
a 18
P 2
237, Let x, y, z > 0 and x + 2xyz = 1. Prove that:
X
3xyz 2
x2 y 2
q
ab
... This ineq becomes:
Let x = (a+c)(b+c)
X
3
a
b+c
2
Xb+ca
b+ca
2
a
b+c
(b + c) 2 b + c a a
so we get:
r
b+ca
b+ca
2
a
b+c
239, Let a, b, c be the length of the sides of triangle ABC. S is the area of ABC and
0 < a < b < c. Prove that:
X
ab(a b) 2
) 4S
(b c)(c a)
4 p 2
a + b2 + c2 S
3
96
(a + b + c)2
(a + b c)(a + c b)(b + c a)
+
4
2(ab + bc + ca) (a2 + b2 + c2 )
abc
x2 + y 2 + z 2
8xyz
+
2
xy + yz + zx (x + y)(y + z)(z + x)
b
c a
ab + bc + ca
ab + bc + ca
LHS 6 +
a2 + b2 + c2
a2 + b2 + c2
ab + bc + ca
+ 2(
+
) 11
ab + bc + ca
ab + bc + ca a2 + b2 + c2
From Cauchy
a b
c
(a + b + c)2
+ +
.
b
c a
ab + bc + ca
Let x = a2 + b2 + c2 , y = ab + bc + ca. Then it remains to show that
3(x + 2y) 2y
+
11
y
x
which in turn is equivalent to
(x y)(3x 2y) 0
which is obviously true since x y.
For your ineq,we can prove easily. For example:
c
2(ab + bc + ca)
2(a2 + b2 + c2 ) 2(ab + bc + ca)
a b
+
+46
2( + + ) +
b
c a
a2 + b2 + c2
ab + bc + ca
a2 + b2 + c2
97
a b
c
8(ab + bc + ca)
17
+ + +
b
c a 3(a2 + b2 + c2 )
3
242, Let a, b, c > 0. Prove that
a2 + bc
3 2
b+c
2
5
a
+ abc
1 + bc
2
Assume a b c
=> C
a
b+c
a
+
+ abc
+ abc + 1 = V
1 + bc 1 + bc
1 + bc
V =
a(bc2 + bc + 1)
+ 1 5/2
1 + bc
P =
1
1
a+b+1
+
a+b+4
a+b+1 3
245, Let x, y, z be positive real numbers such that x2 +y 2 +z 2 12. Find the minimum
of:
y6
z6
x6
p
+
+
S=
xy + 2 1 + z 3
yz + 2 1 + x3
zx + 2 1 + y 3
Its very easy!
We have
p
2 1 + x3 x2 + 2
98
then, use CS
=> min = 96/5
246, Let a, b, c 0. Prove that
Y
(a4 + 7a2 + 10) 216(a + b + c)3
We have :
Y
(a4 + 7a2 + 10) 729(a2 + 1)(b2 + 1)(c2 + 1) 216(a + b + c)3
your solution is not true! We have
(a2 + 2)(b2 + 2)(c2 + 2) 3(a + b + c)2
and my ineq:
(a2 + 5)(b2 + 5)(c2 + 5) 72(a + b + c)
C-S lemma
a4 + 7a2 + 10 6(a3 + 2)
(a3 + 2)(b3 + 2)(c3 + 2) (a + b + c)3
Its very easy C-S.
247, Let a, b, c be positive numbers such that a + b + c = 3. Prove that
abc +
12
5
ab + bc + ac
4X
bc 3
3
12
ab + bc + ca
3
abc +
4(
+
) 3 4.2 3 = 5
ab + bc + ca
3
ab + bc + ca
248, If a, b, c are non-negative numbers, prove that
abc
a2 + ab + b2
3 (a + b)
4
so
a2 + ab + b2
b2 + bc + c2
1
2
2
(a + b + c) (ab + bc + ca) (ab + bc + ca)
3
99
1
(2q)(p+r)(p+r)
2
23
1 (2q) + (p + r) + (p + r) 3
(
) =
= 4.
2
3
2
250, Prove that for all positive real numbres a, b, c
=
ca
ab
bc
+
+
a2 b + a2 c b2 a + b2 c c2 a + c2 b
bc
ac
ab
b+c a+b a+c
1 1 1
+
+
][
+
+
] ( + + )2
a2 (b + c) b2 (a + c) c2 (a + b)
bc
ab
ac
a b
c
252, Let x, y, z > 0 and xyz = 1. Prove that
x2
y2
z2
3
+
+
1+y 1+z
1+x
2
by CS we have
x2
y2
z2
(x + y + z)2
3
+
+
1+y 1+z
1+x
3+x+y+z
2
since for X = x + y + z we have 2X 2 3X 9 0f orX 3.
We can use Cauchy- Schwartz to solve this problem: We have
1+y
x2
+
x
1+y
4
100
c +d
2
1 (a2 + b2 )
c2 + d2
=
2
2
1 (a + b) + ab
1 a2 + b2
2
2
a3
a3
a2
2
3
+ (b + c)
a + b2 + c2
X
a(a3 +(b+c)3 ) = (a+b+c)(
a3 +6abc)
therefore
a2 x + b2 y + c2 z 3xyz
Similarly
ax2 + by 2 + cz 2 3abc
Adding up these inequalities yields the desired rezult.
256, Your inequality is
b
c
a
+
+
1
3 3
3 3
a3 + 26abc
b + 26abc
c + 26abc
By Holder we have
X
3
3
a + 26abc
3 X
X 4
a a3 + 26abc
a
X
ab a2 + b2
+6
X
a2 b2 14abc (a + b + c)
which is true.
257, Let a, b, c be positive real numbers such that a + b + c = 1. Prove that
p
p
p
a(a + bc)
b(b + ca)
c(c + ab)
1
+
+
b + ca
c + ab
a + bc
2 abc
p
X a(a + bc)
LHS
2 abc
but
Xp
p
a(a + bc) (a + b + c)(a + b + c + ab + bc + ca) 2(B C s)
so i get
p
a(a + bc)
1
2 abc
abc
by AM-GM we have :
X
and we have :
X a + a + bc
b + ca
p
a(a + bc)
1 X a + a + bc
b + ca
2
b + ca
X
2a + bc
2a + bc
=
b(a + b + c) + ca
(a + b)(b + c)
102
(2a + bc)(a + c)
(a + b)(a + c)(b + c)
2 abc
(x y)(x z)
cycl
2
(a + b)(a + c)
2
2
2
2
2
+b c +c a
a + b2 + c2
(and similarly for y and z). Note that
x=
a2 b2
(a + b)2
2
(a + c)(b + c)
2
]+(ac)(bc)[ 2 2
]0
a2 b2 + b2 c2 + c2 a2 a2 + b2 + c2
a b + b2 c2 + c2 a2 a2 + b2 + c2
a2 b2
2
(a + b)2
2
0
+ b2 c2 + c2 a2
a + b2 + c2
and
2
(a + c)(b + c)
2
0
a2 b2 + b2 c2 + c2 a2
a + b2 + c2
Here is another solution with AM-GM
We have that
s
ab + bc + ca
(a4 + b4 + c4 )(ab + bc + ca)2
a4 + b4 + c4
+2 2
33 2 2
2
2
2
2
2
2
2
2
a b +b c +c a
a +b +c
(a b + b2 c2 + c2 a2 )(a2 + b2 + c2 )2
103
a b + ca b c + ab c a + bc
3 2
+
+
c+a
a+b
b+c
2
260, Let a, b, c be real numbers. Prove that:
a2 + b2 + c2 ab bc ca 3(a b)(a c)
the inequality becomes
X
3. Prove that:
a2 + b2 + c2 + d2 + ac + bd 3
Okie we will prove
a2 + b2 + c2 + d2 + ac + bd (ad bc) 3
d c 3 2
c d 3 2
) + (b + +
) 0
<=> (a +
2
2
2
2
262, For any three positive reals a, b, c. Prove the inequality
a2 + bc b2 + ca c2 + ab
+
+
a+b+c
b+c
c+a
a+b
Its equivalent to;
a2 + ab + ac + bc b2 + ab + ac + bc c2 + ab + ac + bc
+
+
2(a + b + c)
b+c
a+c
a+b
104
c
a
b
+
+
a+c
b+c
a+b
3
2
Another way:
X
X
a
(a + b + c)2
1
a2
=
=
b+c
(a b + c
(a b + c + b a + c + c b + a
(a b + c + b a + c + c b + a
3
2
(a b + c + b a + c + c b + a
2
3
Equivalent to
(a b + c+b a + c+c b + a = a ab + ac+ b bc + ab+ c ac + bc (2((a + b + c)((ab + bc + ca) =
Since
(a + b + c)2
1
=
3
3
So we have:
r
2
(a b + c + b a + c + c b + a
3
r
X a
1X X 1
1
3 3
3
a
p
=
3
3 2(a + b + c)
2
b+c
b+c
(ab + bc + ca
a
b
+ cb + ac . Prove that
a3 c
b3 a
c3 b
a+b+c
+
+
a b
c
+ + 3
b
c a
265, Let a, b, c be positive real numbers. Prove the inequality
a+b+c
1
1
3
1
+
+
a (b + 1) b (c + 1) c (a + 1)
1 + abc
He showed that:
(1 + abc)
1
1
1
+
+
a (1 + b) b (1 + c) c (1 + a)
+3
3
3
+ 3 abcwhichisof course 6.
3
abc
The inequality is equivalent to
=
X abc + 1
3
a(1 + b)
cyc
X
cyc
abc + ab
1+a
+
)6
a(1 + b)
a(1 + b)
X
1
1
3
1
+
+
X 1+a
b(1 + c)
+
6
a(1 + b)
1+b
true by Am - Gm
x2 + y 2
x+y
xy +
2
2
I saw that somebody else posted a solution while I was typing mine Embarassed . they
are basically the same but so I wouldnt have wrote it for nothing I will post it .
r
x2 + y 2
xy
2
!2
r
x2 + y 2
x2 + y 2
0
+ xy 2 xy
2
2
!2
r
r
x2 + y 2
x2 + y 2
x2 + y 2
(x + y)
+ xy + 2 xy
=
+ xy
2
2
2
r
(x y)2
x2 + y 2
(x y)2
x+y
+ xy
q
2
x+y
x2 +y 2
xy
+
2
r
x+y
2xy
x2 + y 2
+ xy
+
2
x+y
2
r
2xy
x+y
x2 + y 2
(x y)2
(x y)2
+
xy +
q
0
2
2
x+y
2
2
2(x + y)
2( x +y
+
xy)
2
r
r
x2 + y 2
x2 + y 2
2
x+y
xy 0 ( x y) (
xy) 0
2
2
( x y)2 ( x + y)2
q
( x y)2
0 2(x2 + y 2 ) x y 0
2
2
2( x +y
+ xy)
2
2
(x y)2 0
107
(a +
a2 + b2 )3
ab2
2
2
2
2
2
(a + b)
(b + c)
(c + a)
4(a + b + c2 )(ab + bc + ca)2
This inequality follows from
2
1
1
1
a
b
c
(a2 + b2 + c2 )
+
+
+
+
(b + c)2
(c + a)2
(a + b)2
b+c c+a a+b
and
b
c
(a + b + c)2
a
+
+
.
b+c c+a a+b
2(ab + bc + ca)
270, For all nonnegative real numbers a, b and c, no two of which are zero,
X
(a + b + c)4
1
.
2a2 + 3ab + 2b2
7(a2 + b2 + c2 )(ab + bc + ca)2
271,
X
cyc
a2
a(b + c)
cyc
a(b + c)
cyc
X
cyc
1
(a + b + c)4
(b + c)2
1
1
+ 2
ab + bc + ca a + b2 + c2
272, For all nonnegative real numbers a,b and c, no two of which are zero,
p
3 3abc(a + b + c)(a + b + c)2
1
1
1
+
+
(a + b)2
(b + c)2
(c + a)2
4(ab + bc + ca)3
Its obviously trues because of Am-Gm, we have:
X
X
X
(
a2 +
ab)2 .
ab 108witha + b + c = 3
Replacing a, b, cby a1 , 1b , 1c respectively, we have to prove that
p
X a2 b2
3 3(ab + bc + ca)(ab + bc + ca)2
.
(a + b)2
4(a + b + c)3
108
a2 b2
(ab + bc + ca)2
(ab + bc + ca)2
=
2
2
2
2
2
(a + b)
(a + b) + (b + c) + (c + a)
2(a + b2 + c2 + ab + bc + ca)
2(a2 + b2 + c2 + ab + bc + ca)
4(a + b + c)3
or equivalently,
p
2(a + b + c)3 3 3(ab + bc + ca)(a2 + b2 + c2 + ab + bc + ca)
that is
4(a + b + c)6 27(ab + bc + ca)(a2 + b2 + c2 + ab + bc + ca)2
By AM-GM, we see that
27(ab + bc + ca)(a2 + b2 + c2 + ab + bc + ca)2
3
1
2(ab + bc + ca) + (a2 + b2 + c2 + ab + bc + ca) + (a2 + b2 + c2 + ab + bc + ca) =
2
4(a + b + c)6
273,
2a2
1
1
1
2
1
+ 2
+ 2
+ 2
+ bc 2b + ca 2c + ab
ab + bc + ca a + b2 + c2
cyc
X
cyc
2a2
1
4(a + b + c)2
+ bc
which is easy. 274, Let a, b, c > 0 such that a + b + c = 3abc. Prove that
X
1
3
a+b
2
t :
a=
1
1
1
; b = ; c = ; xy + yz + zx = 3
x
y
z
109
LHS =
X xy
xy
3
x+y
2
2
a4
a3
3
b4 + b 2
a(a + b)
+
a2 + 2b2
r
3
b(b + c)
+
b2 + 2c2
r
3
r
c(c + a)
2abc
3
3
c2 + 2a2
a3 + b3 + c3
3
LHS 3 9 2
(a + 2b2 )(b2 + 2c2 )(c2 + a2 )
(a2 + 2b2 )(b2 + 2c2 )(c2 + a2 )
So we need to prove:
(a3 + b3 + c3 )3 abc(a2 + b2 + c2 )3
But
a3 + b3 + c3 3abc
and
3(a3 + b3 + c3 ) (a2 + b2 + c2 )3
277, Let a, b, c be positive reall number satisfyin abc = a + b + c. Prove the following
inequality :
b
c
3 3
a
+
+
1 + a2
1 + b2
1 + c2
4
CM:
X a
3 3
1 + a2
4
110
Setting :
1
1
1
; b = ; c = => xy + yz + zx = 1
x
y
z
a=
x
3
4
3+x
1
3 3
<=>
4
3+x
But its true by Am-Gm.
278, Let a, b, c > 0, ab + bc + ca = 3. Prove that :
a
b
c
+
+
1
5c + 4a
5b + 4c
5a + 4b
Cauchy-schwarz ?
27LHS 2 = [
We need to prove:
(a + b + c)3 9(ab + bc + ca)
But its true by Am-Gm Smile.
279, Given a, b, c > 0. Prove that:
r
3
3
3
a b
c
4 a + b + c
+ + 3.
()
b
c a
3abc
I hope youll like it, my dear friend Smile
This is my solution.
c
a
b
Let = x, = y, = z.
c
a
b
Then we have : xyz = 1 and :
r
1 y
z
x
() x + y + z 3 4 ( + + )
3 z
x y
(x + y + z)4 27(xy 2 + yz 2 + zx2 )
But we have a inequality :
27(xy 2 + yz 2 + zx2 ) 4(x + y + z)3 27
111
y
z
x
;b = ;c =
x
y
z
2
yz
zx
xy
+
+
xy + yz
yz + zx
zx + xy
2
s
X
2(xy + xz + yz)2
xy
1
xy + xz + yz (xy + yz)(x + y + z)2
cyc
But
X
cyc
xy
xy + xz + yz
v
uX
u
2(xy + xz + yz)2
2x(xy + xz + yz)
t
2
(xy + yz)(x + y + z)
(x + z)(x + y + z)2
cyc
x+z
xy + xz + yz
cyc
<=>
<=>
X 2x2 z
x2 + y 2 + z 2
x
+
z
cyc
x2 + y 2 + z 2
x4
y4
z4
2
+ 2
+ 2
2
x + xz
y + xy z + yz
112
x2 + xz
y 2 + xy z 2 + yz
x2 + y 2 + z 2 + xy + xz + yz
2
s
! s
r
X
X
X
X x+y
xy
x2 y
4x2 y
2x2 y
=
=
2
xy + yz
x+z
2
(x + y) (x + z)
(x + y)(x + z)
cyc
cyc
cyc
cyc
Remain to prove that
X
cyc
1
2x2 y
(x + y)(x + z)
2
But
X
cyc
X
2x2 y
1
4x2 y(y + z) (x + y)(x + z)(y + z)(x + y + z)
(x + y)(x + z)
2
cyc
(x3 y + x3 z 2x2 y 2 ) 0.
cyc
4
3
(a + b + c) abc
27
282, Let a + b + c + d = 4 and a, b, c, d 0. Prove that
a2 b + b2 c + c2 a
a2 bc + b2 cd + c2 da + d2 ab 4
It is necessary to prove, that
(a + b + c + d)4 64(a2 bc + b2 cd + c2 da + d2 ab)
If a = min{a, b, c, d} and b = a + x, c = a + y, d = a + z then it is killing. But it is
very ugly.
Let p, q, r, s = a, b, c, d and p q r s. Then by rearrangement inequality,
a2 bc + b2 cd + c2 da + d2 ab = a(abc) + b(bcd) + c(cda) + d(dab)
p(pqr) + q(pqs) + r(prs) + s(qrs) = (pq + rs)(pr + qs)
(
1
pq + rs + pr + qs 2
) = ((p + s)(q + r))2
2
4
1 p+q+r+s 2 2
((
) )
4
2
= 4.
113
4
if a + b + c = 1
27
1 + a2
2
1 + b + c2
X 2x
X 2x2
1 + a2
=
2
1+b+c
y + 2z
xy + 2xz
2x2
2(x + y + z)2
2
xy + 2xz
3(xy + yz + zx)
z
y
z
+
+
2
1 + xy 1 + xz
1 + xy
114
b
c
2a
2b
2c
a
+
+
+
+
= 2
1 + bc 1 + ca 1 + ab
a+b+c a+b+c a+b+c
Other, we have :
(a 1)2 (a + 2)
a(b2 + c2 )
=1
1
2
2
a + abc a +
therefore,
a
b
c
+
+
a2 + b2 + c2 = 1
1 + bc 1 + ca 1 + ab
We are done
a
b
c
+
+
1 + bc 1 + ca 1 + ab
!2
!
!
X a
X
X
a
a(1 + bc)
a 1 + bc
1 + bc
1 + bc
cyc
cyc
cyc
1
(a + b + c)2 =
X
cyc
a
(a + b + c)2
1
1 + bc
a + b + c + 3abc
p2 p 3r 0
a2 = 1. Prove that:
X
a
1
1 + bc
X
X
a
a
a
=
2 +c2 =
b
1a2
1
+
bc
1
+
1
+
cyc
cyc
cyc
2
2
X a(a + 2)(a 1)2
X 2a
1
1
2
=1+
1.
=1+
2
3a
3
3
3(3 a2 )
cyc
cyc
P
P
X a
1X X 1
3 a
6 a
P
P
1 + bc
3
1 + bc
3 + ab
5 + ( a)2
X
115
P
And 1 a 3
P
X
X
X
X
6 a
P 2 1(
a)2 6
a+5 0 (
a1)(
a5) 0, Right
5 + ( a)
by cauchy
X
a
(a2 + b2 + c2 )2
1
P
3
= 3
1 + bc
a + b3 + c3 + a3 bc
a + b3 + c3 + abc
1
1
a3 + b3 + c3 + abc
9
1
P 2
2
+ ab + b
( a)
Solution : We have :
a2 + ab + b2 = (a + b + c)2 (ab + bc + ca) (a + b + c)c
b2 + bc + c2 = (a + b + c)2 (ab + bc + ca) (a + b + c)b
c2 + ca + a2 = (a + b + c)2 (ab + bc + ca) (a + b + c)a
Suppose a + b + c = 1. We have :
()
1
9
1 (ab + bc + ca) a
X
(3c + a + b)2
1
=
a2 + ab + b2
(3c + a + b)2 (a2 + ab + b2 )
cyc
25(a + b + c)2
P
.
(3c + a + b)2 (a2 + ab + b2 )
q
p
9 2
2
x+ x 1=
(x 1) x 1
4
2
2
Let a = x 1, b = x + 1. Then b a = 2. There fore :
2
b a2 = 2
2(a + b) = 9a3
2
b = a2 + 2
2b = 9a3 2a
2
b = a2 + 2
81a6 36a4 8 = 0
2
b = a2 + 2
a2 = 32
2
b = 83
2b = 9a3 2a
5
3
288, Let a, b, c be nonnegative real numbers such that a + b + c = 1. Prove that
X p
a 8b2 + c2 1
x=
X p
3
a 4b2 + c2
4
117
Note that
3b + c
p
3bc
8b2 + c2
2b + c
3abc
1
3
2b + c
a+b+c
a3 + 3abc
bc(b + c)
which is true by Schur.
289,
X
a2 +
ab
X p
a 3b2 + c2
We have
X
X
X 1
X
X
X
X p
2bc
9abc
a(2b+c
)=3
ab2abc
3
ab
a2 +
ab
a 3b2 + c2
b+c
b+c
a+b+c
290,
X p
3
a 4b2 + c2
4
I used two lemmas in my solution
1) 2b + c
2) 8
p
2bc(2b + c)
4b2 + c2
4b2 + 3bc + c2
X
4b2
2b + c
27
2
+ 3bc + c
a+b+c
118
but for the other inequality how to estabilish the right factor Ax + B to write
p
(Ax + B)2 + (4x2 + 1) 2(Ax + B) 4x2 + 1
Thank you very much.
Well, for the second inequality, I dont establish the inequality (since it reverse...)
p
(Ax + B)2 + (4x2 + 1) 2(Ax + B) 4x2 + 1
292, Let a, b, c, d 0 a2 + b2 + c2 + d2 = 4. Prove that:
a3 + b3 + c3 + d3 8
I just have seen that this inequality is too easy.
Squaring the both sides weneed to prove that:
(a3 + b3 + c3 + d3 )2 (a2 + b2 + c2 + d2 )3 .
Using CBS we infer that:
(a3 + b3 + c3 + d3 )2 (a2 + b2 + c2 + d2 )(a4 + b4 + b4 + d4 )
So we need only to prove that:
a4 + b4 + c4 + d4 (a2 + b2 + c2 + d2 )2
We have :
a2 + b2 + c2 + d2 = 4
0 a, b, c, d 2
3
(x y)2 0
294, Prove that for any reals x, y, z which satisfy condition x2 + y 2 + z 2 = 2 we have
x + y + z) xyz + 2
119
We have :
2 = x2 + y 2 + z 2 2yzoryz 1
By Bunhiacopsky we have :
[x(1 yz) + y + z]2 [x2 + (y + z)2 ][(1 yz)2 + 1] (2 + 2yz)(y 2 z 2 2yz + 1) 4
(becauseyz 1) There fore:
|x + y + z xyz| 2
or
x + y + z 2 + xyz
295, If a, b, c > 0. Prove that
b3
c3
a2 + b2 + c2
a3
+
+
b4
c4
(a2 + b2 + c2 )2
(a2 + b2 + c2 )(ab + bc + ca)
a2 + b2 + c2
a4
+
+
=
a(b + c) b(a + c) c(a + b)
2(ab + bc + ca)
2(ab + bc + ca)
2
Proof 1:
I have :
a3
a(b + c)
+
a2
b+c
4
There fore:
b3
c3
ab + bc + ca
a2 + b2 + c2
a3
+
+
a2 + b2 + c2
2
2 cyc ab
c + ab
cyc
120
X a3 (b + c)
a2 + bc
X a3 (b + c)
abc
X b+c
X a3 (b + c)
X1
abc
=
2
2
a + bc
a + bc
a
1
a(b + c)
b(c + a)
c(a + b)
1
1
a
+
b
+
c
+
+
+
+
a2 + bc
b2 + ca
c2 + ab
a
c
b
a2
X a(b + c)
a(b + c)
a2 + bc
!2
X X a(b + c)
(
a)
a2 + bc
X 1
which is equivalent
X (a b)(a c)
2
0
a(a + bc)
which is Vornicu Schur.
From this idea we should square the inequality and then use that
ab(c + a)(c + b)
1
(a2 + bc)(b2 + ca)
for example .Then you will have to prove that
b
(a + b)(a + c) (b + a)(b + c) (c + a)(c + b) X a
+
+
( + )
2
2
2
a + bc
b + ca
c + ab
a
b
Very nice, Ductrung. Here is my solution for it
We have the inequality is equivalent to
X
X a(b + c)
a2 + bc
a(b + c)
a2 + bc
s
+2
!2
X X 1
a
a
X X 1
ab(a + c)(b + c)
a
(a2 + bc)(b2 + ca)
a
121
ab(a + c)(b + c)
3
(a2 + bc)(b2 + ca)
X X 1
+6
a
a2 + bc
a
X a(b + c)
1
c
2
1 2 ab
ab c + ab
1
c
2
1 =
ab c + ab
We are done.
298, For any positive real numbers a, b and c,
s
a(b + c) b(c + a) c(a + b)
1
1 1 1
+ 2
+ 2
27 + (a + b + c)
+ +
a2 + bc
b + ca
c + ab
2
a b
c
299, The Inequality is equivalent to
X a2 (b + c)2
X ab(b + c)(c + a)
15 1 X b + c
+2
+
(a2 + bc)2
(a2 + bc)(b2 + ca)
2
4
a
Notice that
(a2 + bc)(b2 + ca) ab(b + c)(c + a) = c(a + b)(a b)2
then
2
X ab(b + c)(c + a)
6(1)
(a2 + bc)(b2 + ca)
Other hand,
X a2 (b + c)2
(a2 + bc)2
X a2 (b + c)2
4a2 bc
1X
=
4
b c
+ +2
c b
From (1) and (2) we have done! Besides, by the sam ways, we have a nice solution for
an old problem:
s
r
X X 1
X a(b + c)
a
a2 + bc
a
122
2
ab
c2 + ab
0
b+c
X 1
(a + b + c 1)(
)3
b+c
By Am-Gm , we can prove :
LHS (a + b + c 1)
9
2(a + b + c)
+
+
a2 + b2 + c2
a+b
b+c
c+a
2
2
X
Xr a
(a + b + c)(ab + bc + ca)
9
a
)2 2(a+b+c)(
)=4
(
a+b
(a + b)(a + c)
(a + b + c)(ab + bc + ca) abc
2
301, Given a, b, c 0 and a + b + c = 8. Prove that:
268 + 12a2 b2 c2 ab(a b)2 + bc(b c)2 + ca(c a)2
When does equality occurs ?
By "pqr"
4
12(r 1)2 + (q 2 40)2 0
9
302, Given a, b, c > 0 satisfy a + b + c = 3. Prove that
ab bc ca 9abc
21
+
+
+
c
a
b
4
4
Below is my first attempt, which is indirect but fairly short: Rewrite the inequality as
1
1
1
9
21
+ 2+ 2+
a2
b
c
4
4abc
Put x = a1 , y = b1 andz = c1 . Then xy + yz + zx = 3xyz and the inequality
becomes
4(x2 + y 2 + z 2 ) + 9 7(xy + yz + zx)
or after homogenizing
4(x + y + z)2 +
81(xyz)2
15(xy + yz + zx)
(xy + yz + zx)2
123
X a4 + b4
a2
b2
+4
X
a2 b2
p
(
a)2
2
2
2
2
(a + b )(b + c )
Then use
(a2 + b2 )(b2 + c2 ) (a2 + 2b2 + c2 )2
and Cauchy Schwartz. The ineq turns into the form nguoivn gave.
X (a2 + b2 ca cb)2 (a b)2
a2 + b2
b2 + b c2 + c a2 + a
ab + bc + ca + 5
I can prove your problem by one result of hungkhtn and vacs is if a + b + c = 1, a, b, c
be positive numbers then
a2 b + b2 c + c2 a + abc
4
27
a
(a + b + c)2
36(a2 + b2 + c2 )
P 2
P
+b
ab + b a
ab + bc + ca + 5
X
X
X
X
X
4
a2 (
ab2 +
ab) 36
a2 ( abc +
ab)
ab + 5
27
124
4
r + q)(1)
27
2
0
3
1 4q
9
supposing
(1) <=> q + 5 36(1 2q)(
4
4q 1
equality when a = b = c = 31 .
my solution:
b
c
3
a
+
+
b2 + b c2 + c a2 + a
4
a b
c
+ +
b
c a
36(a2 + b2 + c2 )
ab + bc + ca + 5
1
2
; ...
ab ab + bc + ca
Assume:
a b c.
So : Sa ; Sb 0 easyRazz
We have:
Sa + Sc =
a2 (b + c) + c2 (a + b) 2abc
0
abc(ab + bc + ca)
125
a
b
c
3
+
+
) holds
b2 + b c2 + c a2 + a
4
b2 + b
b
b+1
!
r
4
4
Xa
X a
33 a
33 X 4 a
=
aaa
b
b
4
b
44b
r
4
Xa
3Xa
3
33 4 X a
b
4
b
4
b
4(ab + bc + ca)
308, Let a, b, c 0. Prove that
X
X a
a 3
9abc
) +
b+c
(a + b)(b + c)(c + a)
b+c
q = xy + yz + zx, p = x + y + z, r = xyz
my solution
X
X a
a 3
9abc
) +
b+c
(a + b)(b + c)(c + a)
b+c
<=> p3 + 12r + 3pr 16p
<=> p3 + (3r 16)p + 12r 0
f (p) = p3 + (3r 16)p + 12r
f 0 (p) = 3p2 + 3r 16
p
p
f 0 (p) = 0 <=> p = (16 3r)/3 13/3
=> f 0 (p) > 0withp [3; 4]
=> M inf (p) = minf (3)
If p 4
we have
r
16p p3
4p + 9
126
+
+
3
2
b+c
c+a
a+b
By cauchy-swarchz:
X b2 c2
a + b2 c2 b + c2 a2 c + a2 b2
(a + b + c)2
a + b + c X b2 c2
+
+
+
b+c
c+a
a+b
2(ab + bc + ca)
b+c
2
b+c
By Am-Gm We have:
X a2 b2
X
a+b
(
+
)
ab = 3
a+b
4
By Am-Gm and Cauchy Schwarts, we can prove easily the stronger:
5(a2 + b2 + c2 ) 1
a + b2 c2
b + c2 a2
c + a2 b2
+
+
+
6
2
b+c
c+a
a+b
The first, using my old result:
X a
1
a2 + b2 + c2
+
ab + bc + ca 2
b+c
Besides, by CS and Am-Gm:
s
r
X a2 b2
X
X a2 b2
X ab X a2 + b2 + c2
1 2
a2 b2 .
.(a + b2 + c2 )2 .
=
2
a+b
(a + b)
3
4
2
Add 2 inequalities, we have my stronger
310, Let x, y, z be postive real numbers such that xyz = x + y + z + 2. Prove that:
2( xy + yz + zx) x + y + z + 6
Proof: The inequality is enquivalent to:
X
X
p
(
x)2 (x + y + z) x + y + z + 6or
x 2(x + y + z + 3)
Denote
c+a
a+b
b+c
,y =
,z =
a
b
c
Therefore, we just need to prove:
r
r
r
r
X b+c
Xb+c
X
1 1 1
1 1 1
2(
+ 3) = 2(a + b + c)( + + ) = [
(b + c)]( + + )
a
a
a b
c
a b
c
x=
But this is obviously true due to Cauchy-Schwartz, which ends our proof.
311, Given a, b, c are prositive real numbers a2 + b2 + c2 = 1. Find max of P:
P =
ab
bc
ca
+
+
2
2
1+c
1+a
1 + b2
127
c2
X
ab
ab
1X
b2
a2
p
+ 2
)
( 2
2
2
2
+b +c
4
a +c
c + b2
2 (a2 + c2 )(b2 + c2 )
X
ab
(a + b)
b2
1X
a2
+
)
(
a2 + c2 + b2 + c2
4(a2 + c2 + b2 + c2 )
4
a2 + c2
c2 + b2
1
1
1
+ 2 + 2 14
2
a
b
c
Setting :
1
1
1
1
1
1
= x + 1;
+
= 2 + y; +
+
= z + 3(x, y, z 0)
3c
2b 3c
a 2b 3c
1
1
1
+ 2 + 2 14 = 2(z + 3y + 5x) + (x y)2 + (y z)2 + x2 0
a2
b
c
Then we have done Mr. It holds when x = y = z = 0
=>
=> a = 1; b =
1
1
;c =
2
3
We have
1
1
1 1 1
1
1
1
1
1
1
a + 2b +
+
+
=
(
+
+
)+3(
+
)+5(
)
3(
3
a2 b2 c2
a2 4b2 9c2
4b2 9c2
9c2
1
3c 2
) +6( 2b
+
3
1
3c 2
) +5(
1 2
) 3+6+5 =
3c
(x y)2 0
a3 + b3 + c3
15 3 abc
+
1
(a + b)(b + c)(c + a) 8(a + b + c)
317, Let a, b, c be positive real numbers such that a + b + c = 3. Prove that
1 1 1
8
+ +
+ 9 10(a2 + b2 + c2 ).
a b
c
Equality holds for a = 2, b = c = 12 .
Setting:
a + b + c = p = 3; ab + bc + ca =
8.
p2 t 2
; abc = r
3
9 t2
+ 27 10(2t2 + 9) 0(3 t 0)
r
(3 t)2 (3 + 2t)
.
27
129
1
1
1
, b := , c :=
a
b
c
+5
a b
c
abc(a + b + c)
(ab + bc + ca)(a + b + c)2 4(a2 b2 + b2 c2 + c2 a2 ) + 5abc(a + b + c)
ab(a b)2 + bc(b c)2 + ca(c a)2 0
This proof also shows that the best constant is k = 4. So, your statement is valid, shaam.
We can also prove it by Muirhead inequality: Our inequality is equivolent to
(a + b + c)(ab + ac + bc)2 4abc(a2 + b2 + c2 ) + 5abc(ab + ac + bc)
X
X
X
X
X
a3 b2 + 5
a2 b2 c +
a3 bc 2
a3 bc + 5
a2 b2 c
sym
sym
cyclic
a3 b2
sym
sym
cyclic
a3 bc
sym
2
1
...
bc ab + bc + ca
(a + b + c)2
a2 + b2 + c2
+5=4
3
ab + bc + ac
ab + bc + ac
X a
X (a + b)2
b
(a + b + c)2
1 1 1
( + + 2) =
4
(a + b + c)( + + ) + 3 =
a b
c
b
a
ab
ab + bc + ac
cyc
cyc
4
How about
(a + b 2c)2 (b + c 2a)2 (c + a 2b)2 0
Dear Zaizai, i was asking how to transform it. Because:
(a b)2 (b c)2 (c a)2
130
cyc
cyc
a 3
b 3
c 3
abc
1 a2 + b2 + c2 2
) +(
) +(
) +
.(
)
b+c
c+a
a+b
(a + b)(b + c)(c + a)
2 ab + bc + ca
b 3
c 3
5abc
a2 + b2 + c2
a 3
) +(
) +(
) +
b+c
c+a
a+b
(a + b)(b + c)(c + a)
ab + bc + ca
PS: With this old ineq, we are easily to solve it by Am-Gm and... Notice that:
abc(a2 + b2 + c2 ab bc ca) 0
I havent tried with AM-GM as you said but the inequality is actually weak and we can
prove it using Cauchy Schwarz + Schur as follow: Applying Cauchy Schwarz Inequality, we have
b
c
(a + b + c)2
a
+
+
a
b
c
+
+
1
b+c c+a a+b
2a
b+c , y
2b
c+a , z
2c
a+b ,
2
a
b
c
+
+
1
b+c c+a a+b
131
p(4qp2 )
,
9
It follows that
q
and we obtain
r =4q 4
p3 + 36
4p + 9
p3 + 36
p(16 p2 )
=
4p + 9
4p + 9
We have to prove
(p 4)(p2 + 4) + (3p + 4)
p(16 p2 )
0
4p + 9
b 3
c 3
5abc
a2 + b2 + c2
a 3
) +(
) +(
) +
b+c
c+a
a+b
(a + b)(b + c)(c + a)
ab + bc + ca
I havent read Thuans book, so I dont know that solution. Anyways, the following
inequality is stronger than yours Wink
3
3
3
a
b
c
b
c
9abc
a
+
+
.
+
+
+
b+c
c+a
a+b
(a + b)(b + c)(c + a)
b+c c+a a+b
Your proof is same to me Embarassed. I also used a nice lenma:
X
a
a2 + b2 + c2
4abc
+
b+c
ab + bc + ca (a + b)(b + c)(c + a)
b+c
2(ab + bc + ca)
a2 + b2 + c2 +
4abc(ab + bc + ca)
(a + b)(b + c)(c + a)
1
4abc(ab + bc + ca)
a2 + b2 + c2 +
b+c
(a + b)(b + c)(c + a)
132
1
4(ab + bc + ca)
b+c
(a + b)(b + c)(c + a)
1
9
b+c
2(a + b + c)
4(ab + bc + ca)
9
(a + b)(b + c)(c + a)
2(a + b + c)
I meant:
a
a2 + b2 + c2
4abc
+
b+c
ab + bc + ca (a + b)(b + c)(c + a)
(a + b + c)(a2 + b2 + c2 ) abc
a2 + b2 + c2
(a + b)(b + c)(c + a)
ab + bc + ca
X
<=> (a+b+c)(a2 +b2 +c2 )(ab+bc+ca)abc
ab [(a+b+c)(ab+bc+ca)abc](a2 +b2 +c2 )
<=>
=
,
a + b + c a + b + 7c
(a + b + c)(a + b + 7c)
it suffices to prove that
X
ca
(a + b + c)(ab + bc + ca)
a + b + 7c
9(a2 + b2 + c2 )
ca
(ab + bc + ca)2
P
.
a + b + 7c
ca(a + b + 7c)
a3 b + 4
ab3 4
133
a2 b +
a2 b2 abc
a,
ab2 + 3abc),
that is
a.
ab(a 2b)2
i2
i X h X
abc(a 2b) = abc(a + b + c)2 ,
c
hence
a+b+c+ + +
a
b
c
3
a b
c
Its easy
1
1
1
(a + ) b +
c+
4(c +
a
b
c
1
1
1
(a + ) b +
c+
4(b +
a
b
c
1
1
1
(a + ) b +
c+
4(a +
a
b
c
1
)
c
1
)
b
1
)
a
a+b+c+ + +
a
b
c
3
a b
c
324, Let a, b, c > 0. Prove that
(1 +
ab)2
(1 +
bc)2
(1 +
ca)2
3+a+b+c
(1 + a)(1 + b)(1 + c)
ab)2 (1 + a)(1 + b)
X
1
1
1
1
3+a+b+c
2
+
+
=
2
2
(1
+
a)(1
+
b)
(1
+
a)(1 + b)(1 + c)
(1
+
ca)
(1 + ab) (1 + bc)
(1 +
134